You Answered Trans Metatarsal Amputation

You might also like

Download as docx, pdf, or txt
Download as docx, pdf, or txt
You are on page 1of 98

Theme: Amputations

A. Transfemoral amputation
B. Gritti - Stokes amputation
C. Digital amputation
D. Syme's amputation
E. Hindquarter amputation
F. Below knee amputation
G. Trans metatarsal amputation
H. Amputation of digit

Please select the most appropriate procedure for the scenario given. Each option may
be used once, more than once or not at all.

1. The operation of choice for a 90 year old lady with infected gangrene of the mid
foot secondary to diabetes. She has fixed flexion deformity of the knee.

You answered Trans metatarsal amputation

The correct answer is Transfemoral amputation

An elderly patient with diabetes and peripheral vascular disease is a high risk
surgical candidate. It is important that the chances of a successful outcome are
maximised at the first operation. SInce above knee amputations usually heal
more reliably than below knee amputations this is a preferable option, especially
since she has a fixed deformity.

2. An operation in which Skew flaps are created.

You answered Gritti - Stokes amputation

The correct answer is Below knee amputation

This is one variant of a below knee amputation. The Burgess flap is the other
commonly practised approach.

3. An amputation of the lower limb in which the femoral condyles are removed
and the patella retained.

You answered Transfemoral amputation

The correct answer is Gritti - Stokes amputation

This is a Gritti - Stokes amputation. During a Gritti - Stokes operation the


patella is conserved and swung posteriorly to cover the distal femoral surface.

Beware performing amputations in


patients with peripheral vascular disease
without optimising inflow first!

Amputations

Amputations are indicated when the affected limb is one of the following:

 Dead non viable


 Deadly where it is posing a major threat to life
 Dead useless where it is viable but a prosthesis would be preferable

Orthopaedic surgery

 Amputation is often undertaken as an option of last resort e.g. Limb salvage


has failed and the limb is so non functional that mobility needs would be best
met with prosthesis.
 Chronic fracture non union or significant limb shortening following trauma
would fit into this category. Occasionally following major trauma a primary
amputation is preferable. This would be the case in an open fracture with
major distal neurovascular compromise and other more life threatening
injuries are present.

Vascular surgery

 The first two categories are the most prevalent.


 Diabetic foot sepsis is often a major cause of sepsis which can spread rapidly
in the presence of established peripheral vascular disease.
 As a general rule the main issue in vascular surgery is to optimise vascular
inflow prior to surgery. The more distal the planned amputation is to be, the
more important this rule becomes.
 In other situations there has been something such as an embolic event that has
not been revascularised in time. In this case the limb shows fixed mottling and
an amputation will be needed.

Types of amputations
As the vast majority of commonly performed amputations affect the lower limbs these
will be covered here.

The main categories of amputations are:


 Pelvic disarticulation (hindquarter)
 Above knee amputation
 Gritti Stokes (through knee amputation)
 Below knee amputation (using either Skew or Burgess flaps)
 Syme's amputation (through ankle)
 Amputations of mid foot and digits

Choosing a level of amputation depends on:

 The disease process being treated


 Desired functional outcome
 Co-morbidities of the patient

Above knee amputations

 Quick to perform
 Heal reliably
 Patients regain their general health quickly
 For this benefit, a functional price has to be paid and many patients over the
age of 70 will never walk on an above knee prosthesis.
 Above knee amputations use equal anterior-posterior flaps

Below knee amputations

 Technically more challenging to perform


 Heal less reliably than their above knee counterparts.
 However, many more patients are able to walk using a below knee prosthesis.
 In below knee amputations the two main flaps are Skew flaps or the Burgess

Long posterior flap. There is some evidence that Skew flaps are better
vascularised than the long posterior flap and some vascular surgeons prefer
them for this reason.

It is worth remembering that whilst it may be technically feasible to offer a below


knee amputation there may be circumstances where an above knee option is
preferable. For example, in fixed flexion deformities of the lower limb, little
functional benefit would be gained from below knee amputation surgery.
Theme: Vascular disorders affecting the upper limb

A. Proximal brachial artery occlusion secondary to atheroma


B. Distal brachial artery occlusion secondary to atheroma
C. Axillary artery embolus
D. Axillary vein thrombosis
E. Cervical rib
F. Raynaud's disease
G. Rheumatoid disease

Please select the most likely cause for the presenting scenario described. Each option
may be used once, more than once or not at all.

4. A 73 year old male presents with a collapse and is brought to the emergency
department. On examination he has a cold, painful left hand and forearm.

You answered Distal brachial artery occlusion secondary to atheroma

The correct answer is Axillary artery embolus

Theme from September 2012 Exam


Sudden arterial embolus will affect the axillary artery in up to 30% cases.
Because of the acute nature of the condition there is not time for the
development of a collateral circulation so the limb is usually pale and painful.
Emboli occur usually occur as a result of atrial fibrillation. Fast atrial fibrillation
can cause syncope and an acute embolus.

5. A 23 year old man presents with intermittent symptoms of altered sensation in


his arm and discomfort when he uses his hands. He works as an electrician and
his symptoms are worst when he is fitting light fixtures.

Cervical rib

Compression of the thoracic outlet by the fibrous band of the "rib" can result in
both neurological and circulatory compromise. When manual tasks are
performed in which the hand works overhead the signs and symptoms will be
maximal and this is the basis of Adsons test.

6. A 19 year old lady presents with recurrent episodes of pain in her hands. She
notices that her symptoms are worst in cold weather. When she gets the pain she
notices that her hands are very pale, they then become dark blue in colour.

Raynaud's disease

Raynauds disease is characterised by a series of colour changes and discomfort


is often present. The young age at presentation coupled with the absence of a
smoking history (in most cases) makes occlusive disease unlikely.

Vascular disorders of the upper limb


Upper limb arterial disease is less common than lesions causing symptoms in the
lower limb. The upper limb circulation may be affected by embolic events, stenotic
lesions (both internal and extrinsic), inflammatory disorders and venous diseases.
The anatomy of the collateral circulation of the arterial inflow may impact on the
history and nature of disease presentation. In the region of the subclavian and axillary
arteries the collateral vessels passing around the shoulder joint may provide pathways
for flow if the main vessels are stenotic or occluded. During periods of increased
metabolic demand the collateral flow is not sufficient and the vertebral arteries may
have diminished flow. This may result in diminished flow to the brain with
neurological sequelae such as syncope.

Vascular disease of the upper limb


Condition Features
Axillary/  50% of upper limb emboli will lodge in the brachial artery
brachial  30% of upper limb emboli will lodge in the axillary artery
embolus  Sudden onset of symptoms; pain, pallor, paresis, pulselessness,
paraesthesia
 Sources are left atrium with cardiac arrhythmia (mainly AF),
mural thrombus
 Cardiac arrhythmias may cause result in impaired
consciousness in addition to the embolus

Arterial  Those resulting from atheroma are the most common, trauma
occlusions may result in vascular changes and long term occlusion but this
is rare
 Features may include claudication, ulceration and gangrene.
Proximally sited lesions may result in subclavian steal
syndrome
 The progressive nature of the disease allows development of
collaterals, acute ischaemia may occur as a result of acute
thrombosis

Raynaud's  Idiopathic condition affecting young females


disease  Usually affects hands > feet
 Digits become: white -->blue -->red
 Treatment is with calcium antagonists

Upper limb  Gradual onset of upper limb swelling and discomfort.


venous  Sensation and motor function are normal
thrombosis  Condition may complicate pre-existing malignancy (especially
breast cancer) or arise as a result of repetitive use of the limb in
a task such as painting a ceiling
 The condition is diagnosed with duplex ultrasound and
treatment is with anticoagulation

Cervical rib  0.2-0.4% incidence


 Consist of an anomalous fibrous band that often originates from
C7 and may arc towards, but rarely reaches the sternum
 Congenital cases may present around the third decade, some
cases are reported to occur following trauma
 Bilateral in up to 70%
 Compression of the subclavian artery may produce absent
radial pulse on clinical examination and in particular may result
in a positive Adsons test (lateral flexion of the neck away from
symptomatic side and traction of the symptomatic arm- leads to
obliteration of radial pulse)
 Treatment is most commonly undertaken when there is
evidence of neurovascular compromise. A transaxillary
approach is the traditional operative method for excision

Theme: Management of occlusive vascular disease

A. Aorto-bifemoral bypass graft


B. Femoro-femoral cross over graft
C. Femoro-popliteal bypass graft
D. Femoro-distal bypass graft
E. Axillo-bifemoral bypass graft
F. Bilateral above knee amputation

Please select the most appropriate arterial bypass method for the scenario described.
Each option may be used once, more than once or not at all.

7. An 83 year old lady with a significant cardiac history is admitted with rest pain
and bilateral leg ulcers. Imaging demonstrates bilateral occlusion of both
common iliac arteries that are unsuitable for stenting.

Axillo-bifemoral bypass graft

Theme from January 2012 Exam

In patients with major cardiac co-morbidities the safest option is to choose an


axillo-bifemoral bypass graft. The long term patency rates are less good than
with aorto-bifemoral bypass grafts, however, the operation is less major.

8. A 54 year old man presents to the vascular clinic with severe rest pain and an
ulcer on his right foot that is not healing. On examination he has bilateral absent
femoral pulses. Imaging demonstrates a bilateral occlusion of the common iliac
arteries that is not suitable for stenting.

Aorto-bifemoral bypass graft

In a young patient consideration should be given to aorto-bifemoral bypass


grafts as these have the best long term functional outcome compared with an
axillobifemoral bypass graft.

9. A 78 year old man presents with left sided rest pain in his leg and a non healing
arterial leg ulcer on the same leg. Imaging shows normal right leg vessels, on
the left side there is a long occlusion of the external iliac artery that is unsuitable
for stenting. He has a significant cardiac history.

Femoro-femoral cross over graft

Femoro-femoral cross over grafts are an option for treatment of iliac occlusions
in patients with significant co-morbidities and healthy contralateral vessels. In
reality the idealised situation presented here seldom applies and the opposite
vessels usually have some disease and one must be careful not to damage the
"healthy" side.

Peripheral vascular disease

Indications for surgery to revascularise the lower limb

 Intermittent claudication
 Critical ischaemia
 Ulceration
 Gangrene

Intermittent claudication that is not disabling may provide a relative indication, whilst
the other complaints are often absolute indications depending upon the frailty of the
patient.

Assessment

 Clinical examination
 Ankle brachial pressure index measurement
 Duplex arterial ultrasound
 Angiography (standard, CT or MRI): usually performed only if intervention
being considered.

Angioplasty
In order for angioplasty to be undertaken successfully the artery has to be accessible.
The lesion relatively short and reasonable distal vessel runoff. Longer lesions may be
amenable to sub-intimal angioplasty.

Surgery
Surgery will be undertaken where attempts at angioplasty have either failed or are
unsuitable. Bypass essentially involves bypassing the affected arterial segment by
utilising a graft to run from above the disease to below the disease. As with
angioplasty good runoff improves the outcome.

Some key concepts with bypass surgery

Superficial femoral artery occlusion to the above knee

 Angioplasty may be attempted but otherwise these patients will require a


femoro-popliteal bypass graft.
 Patency rates for Polytetrafluoroethylene (PTFE) and vein are similar, so
PTFE preferred unless co-existing infection makes use of prosthetic material
undesirable.

Procedure

 Artery dissected out, IV heparin 3,000 units given and then the vessels are
cross clamped
 Longitudinal arteriotomy
 Graft cut to size and tunneled to arteriotomy sites
 Anastomosis to femoral artery usually with 5/0 'double ended' Prolene suture
 Distal anastomosis usually using 6/0 'double ended' Prolene

Distal disease

 Femoro-distal bypass surgery takes longer to perform, is more technically


challenging and has higher failure rates.
 In elderly diabetic patients with poor runoff a primary amputation may well be
a safer and more effective option. There is no point in embarking on this type
of surgery in patients who are wheelchair bound.
 In femorodistal bypasses vein gives superior outcomes to PTFE.

Rules

 Vein mapping 1st to see whether there is suitable vein (the preferred conduit).
Sub intimal hyperplasia occurs early when PTFE is used for the distal
anastomosis and will lead to early graft occlusion and failure.
 Essential operative procedure as for above knee fem-pop.
 If there is insufficient vein for the entire conduit then vein can be attached to
the end of the PTFE graft and then used for the distal anastomosis. This type
of 'vein boot' is technically referred to as a Miller Cuff and is associated with
better patency rates than PTFE alone.
 Remember the more distal the arterial anastomosis the lower the success rate.
References
Peach G et alDiagnosis and management of peripheral arterial disease. BMJ 2012;
345: 36-41.
Theme: Management of occlusive vascular disease

A. Aorto-bifemoral bypass graft


B. Femoro-femoral cross over graft
C. Femoro-popliteal bypass graft
D. Femoro-distal bypass graft
E. Axillo-bifemoral bypass graft
F. Bilateral above knee amputation

Please select the most appropriate arterial bypass method for the scenario described.
Each option may be used once, more than once or not at all.

7. An 83 year old lady with a significant cardiac history is admitted with rest pain
and bilateral leg ulcers. Imaging demonstrates bilateral occlusion of both
common iliac arteries that are unsuitable for stenting.

Axillo-bifemoral bypass graft

Theme from January 2012 Exam

In patients with major cardiac co-morbidities the safest option is to choose an


axillo-bifemoral bypass graft. The long term patency rates are less good than
with aorto-bifemoral bypass grafts, however, the operation is less major.

8. A 54 year old man presents to the vascular clinic with severe rest pain and an
ulcer on his right foot that is not healing. On examination he has bilateral absent
femoral pulses. Imaging demonstrates a bilateral occlusion of the common iliac
arteries that is not suitable for stenting.

Aorto-bifemoral bypass graft

In a young patient consideration should be given to aorto-bifemoral bypass


grafts as these have the best long term functional outcome compared with an
axillobifemoral bypass graft.

9. A 78 year old man presents with left sided rest pain in his leg and a non healing
arterial leg ulcer on the same leg. Imaging shows normal right leg vessels, on
the left side there is a long occlusion of the external iliac artery that is unsuitable
for stenting. He has a significant cardiac history.
Femoro-femoral cross over graft

Femoro-femoral cross over grafts are an option for treatment of iliac occlusions
in patients with significant co-morbidities and healthy contralateral vessels. In
reality the idealised situation presented here seldom applies and the opposite
vessels usually have some disease and one must be careful not to damage the
"healthy" side.

Peripheral vascular disease

Indications for surgery to revascularise the lower limb

 Intermittent claudication
 Critical ischaemia
 Ulceration
 Gangrene

Intermittent claudication that is not disabling may provide a relative indication, whilst
the other complaints are often absolute indications depending upon the frailty of the
patient.

Assessment

 Clinical examination
 Ankle brachial pressure index measurement
 Duplex arterial ultrasound
 Angiography (standard, CT or MRI): usually performed only if intervention
being considered.

Angioplasty
In order for angioplasty to be undertaken successfully the artery has to be accessible.
The lesion relatively short and reasonable distal vessel runoff. Longer lesions may be
amenable to sub-intimal angioplasty.

Surgery
Surgery will be undertaken where attempts at angioplasty have either failed or are
unsuitable. Bypass essentially involves bypassing the affected arterial segment by
utilising a graft to run from above the disease to below the disease. As with
angioplasty good runoff improves the outcome.

Some key concepts with bypass surgery

Superficial femoral artery occlusion to the above knee


 Angioplasty may be attempted but otherwise these patients will require a
femoro-popliteal bypass graft.
 Patency rates for Polytetrafluoroethylene (PTFE) and vein are similar, so
PTFE preferred unless co-existing infection makes use of prosthetic material
undesirable.

Procedure

 Artery dissected out, IV heparin 3,000 units given and then the vessels are
cross clamped
 Longitudinal arteriotomy
 Graft cut to size and tunneled to arteriotomy sites
 Anastomosis to femoral artery usually with 5/0 'double ended' Prolene suture
 Distal anastomosis usually using 6/0 'double ended' Prolene

Distal disease

 Femoro-distal bypass surgery takes longer to perform, is more technically


challenging and has higher failure rates.
 In elderly diabetic patients with poor runoff a primary amputation may well be
a safer and more effective option. There is no point in embarking on this type
of surgery in patients who are wheelchair bound.
 In femorodistal bypasses vein gives superior outcomes to PTFE.

Rules

 Vein mapping 1st to see whether there is suitable vein (the preferred conduit).
Sub intimal hyperplasia occurs early when PTFE is used for the distal
anastomosis and will lead to early graft occlusion and failure.
 Essential operative procedure as for above knee fem-pop.
 If there is insufficient vein for the entire conduit then vein can be attached to
the end of the PTFE graft and then used for the distal anastomosis. This type
of 'vein boot' is technically referred to as a Miller Cuff and is associated with
better patency rates than PTFE alone.
 Remember the more distal the arterial anastomosis the lower the success rate.

References
Peach G et alDiagnosis and management of peripheral arterial disease. BMJ 2012;
345: 36-41.
Theme: Ankle brachial pressure index

A. >1.2
B. 1.0
C. 0.8
D. 0.5
E. 0.3

Please select the ankle brachial pressure index that is most likely to be present for the
scenario given. Each value may be used once, more than once or not at all.

10. A 73 year old lifelong heavy smoker presents to the vascular clinic with
symptoms of foot ulceration and rest pain. On examination her foot has areas
of gangrene and pulses are impalpable.

0.3

This is critical limb ischaemia. Values of 0.3 are typical in this setting and
urgent further imaging is needed. Debridement of necrosis prior to improving
arterial inflow carries a high risk of limb loss.

11. A 63 year old man presents with a claudication distance of 15 yards. He is a


lifelong heavy smoker. On examination his foot is hyperaemic and there is a
small ulcer at the tip of his great toe.

You answered >1.2

The correct answer is 0.5

Hyperaemia may occur in association with severe vascular disease and is


referred to surgically as a "sunset foot". ABPI is usually higher than 0.3, but
seldom greater than 0.5. Especially when associated with hyperaemic changes
and ulceration. Urgent further imaging and risk factor modification is needed.

12. A 77 year old morbidly obese man with type 2 diabetes presents with leg pain
at rest. His symptoms are worst at night and sometimes improve during the
day. He has no areas of ulceration.

You answered 0.8

The correct answer is >1.2

Type 2 diabetes may have vessel calcification. This will result in abnormally
high ABPI readings. Pain of this nature in diabetics is usually neuropathic and
if a duplex scan is normal then treatment with an agent such as carbamazepine
is sometimes helpful.

Theme from September 2011 and


September 2012 exam
Ankle-Brachial pressure index

 Measurement of ankle- brachial pressure index (ABPI) is a commonly


performed vascular investigation.
 Calculated by dividing lower limb pressure by the highest upper limb pressure.

Results of ABPI
1.2 or greater Usually due to vessel calcification
1.0- 1.2 Normal
0.8-1.0 Minor stenotic lesion
Initiate risk factor management
0.50-0.8 Moderate stenotic lesion
Consider duplex
Risk factor management
If mixed ulcers present then avoid tight compression bandages
0.5- 0.3 Likely significant stenosis
Duplex scanning to delineate lesions needed
Compression bandaging contra indicated
Less than 0.3 Indicative of critical ischaemia
Urgent detailed imaging required
A 67 year old patient is due to undergo a femoro-popliteal bypass graft. Which
heparin regime should the surgeon ask for prior to cross clamping the femoral artery?

A. Single therapeutic dose of low molecular weight heparin on the ward


prior to coming to theatre

B. Single therapeutic dose of low molecular weight heparin the night


before surgery

C. Dose of 10,000 units of unfractionated heparin prior to induction of


anaesthesia

D. Dose of 3,000 units of unfractionated heparin, 3 minutes prior to cross


clamping

E. Dose of 30,000 units of unfractionated heparin, 3 minutes prior to


cross clamping

As a rule most vascular surgeons will administer approximately 3,000 units of


systemic heparin 3-5 minutes prior to cross clamping to help prevent further intra
arterial thromboses. A dose of 30,000 units is given prior to going on
cardiopulmonary bypass. Heparin given at induction will cause bleeding during
routine dissection.

Heparin
 Causes the formation of complexes between antithrombin and activated
thrombin/factors 7,9,10,11 & 12

Advantages of low molecular weight heparin

 Better bioavailability
 Lower risk of bleeding
 Longer half life
 Little effect on APTT at prophylactic dosages
 Less risk of HIT

Complications

 Bleeding
 Osteoporosis
 Heparin induced thrombocytopenia (HIT): occurs 5-14 days after 1st exposure
 Anaphylaxis

In surgical patients that may need a rapid return to theatre administration of


unfractionated heparin is preferred as low molecular weight heparins have a longer
duration of action and are harder to reverse.
An 18 year old lady presents with extensive varicose veins of her left leg.
There is associated port wine staining. What is the most likely diagnosis?

A. Type 1 diabetes

B. Osler syndrome

C. Gardner's syndrome

D. Proteus syndrome

E. Klippel-Trenaunay-Weber syndrome

A less common cause of venous insufficiency is Klippel-Trenaunay-Weber (KTW)


syndrome, which involves port-wine stains, varicose veins, and bony or soft-tissue
hypertrophy.

Lower leg ulcers

Venous leg ulcers

 Most due to venous hypertension, secondary to chronic venous insufficiency


(other causes include calf pump dysfunction or neuromuscular disorders)
 Ulcers form due to capillary fibrin cuff or leucocyte sequestration
 Features of venous insufficiency include oedema, brown pigmentation,
lipodermatosclerosis, eczema
 Location above the ankle, painless
 Deep venous insufficiency is related to previous DVT and superficial venous
insufficiency is associated with varicose veins
 Doppler ultrasound looks for presence of reflux and duplex ultrasound looks at
the anatomy/ flow of the vein
 Management: 4 layer compression banding after exclusion of arterial disease
or surgery
 If fail to heal after 12 weeks or >10cm2 skin grafting may be needed

Marjolin's ulcer

Image sourced from Wikipedia

 Squamous cell carcinoma


 Occurring at sites of chronic inflammation e.g; burns, osteomyelitis after 10-
20 years
 Mainly occur on the lower limb

Arterial ulcers

 Occur on the toes and heel


 Painful
 There may be areas of gangrene
 Cold with no palpable pulses
 Low ABPI measurements
Neuropathic ulcers

 Commonly over plantar surface of metatarsal head and plantar surface of


hallux
 The plantar neuropathic ulcer is the condition that most commonly leads to
amputation in diabetic patients
 Due to pressure
 Management includes cushioned shoes to reduce callous formation

Pyoderma gangrenosum

Image sourced from Wikipedia

 Associated with inflammatory bowel disease/RA


 Can occur at stoma sites
 Erythematous nodules or pustules which ulcerate

Theme: Management of peripheral arterial disease

A. Primary amputation
B. Angioplasty
C. Arterial bypass surgery using vein
D. Arterial bypass surgery using PTFE
E. Conservative management with medical therapy and exercise
F. Watch and wait
G. Duplex scanning

Please select the most appropriate management for the scenario given. Each option
may be used once, more than once or not at all.

15. A 63 year old man is admitted with rest pain and foot ulceration. An
angiogram shows a 3 cm area of occlusion of the distal superficial femoral
artery with 3 vessel run off. His ankle - brachial pressure index is 0.4.

You answered Arterial bypass surgery using vein

The correct answer is Angioplasty

Short segment disease and good run off with tissue loss is a compelling
indication for angioplasty. He should receive aspirin and a statin if not already
taking them.

16. A 72 year old man present in the vascular clinic with calf pain present on
walking 100 yards. He is an ex-smoker and lives alone. On examination he has
reasonable leg pulses. His right dorsalis pedis pulse gives a monophasic
doppler signal with an ankle brachial pressure index measurement of 0.7. All
other pressures are acceptable. There is no evidence of ulceration or gangrene.

Conservative management with medical therapy and exercise

Structured exercise programmes combined with medical therapy will improve


many patients. Should his symptoms worsen or fail to improve then imaging
with duplex scanning would be required.

17. An 83 year old lady is admitted from a nursing home with infected lower leg
ulcers. She underwent an attempted long superficial femoral artery sub initimal
angioplasty 2 weeks previously. This demonstrated poor runoff below the
knee.

Primary amputation

Poor runoff and sepsis would equate to poor outcome with attempted bypass
surgery.

Peripheral vascular disease

Indications for surgery to revascularise the lower limb

 Intermittent claudication
 Critical ischaemia
 Ulceration
 Gangrene
Intermittent claudication that is not disabling may provide a relative indication, whilst
the other complaints are often absolute indications depending upon the frailty of the
patient.

Assessment

 Clinical examination
 Ankle brachial pressure index measurement
 Duplex arterial ultrasound
 Angiography (standard, CT or MRI): usually performed only if intervention
being considered.

Angioplasty
In order for angioplasty to be undertaken successfully the artery has to be accessible.
The lesion relatively short and reasonable distal vessel runoff. Longer lesions may be
amenable to sub-intimal angioplasty.

Surgery
Surgery will be undertaken where attempts at angioplasty have either failed or are
unsuitable. Bypass essentially involves bypassing the affected arterial segment by
utilising a graft to run from above the disease to below the disease. As with
angioplasty good runoff improves the outcome.

Some key concepts with bypass surgery

Superficial femoral artery occlusion to the above knee

 Angioplasty may be attempted but otherwise these patients will require a


femoro-popliteal bypass graft.
 Patency rates for Polytetrafluoroethylene (PTFE) and vein are similar, so
PTFE preferred unless co-existing infection makes use of prosthetic material
undesirable.

Procedure

 Artery dissected out, IV heparin 3,000 units given and then the vessels are
cross clamped
 Longitudinal arteriotomy
 Graft cut to size and tunneled to arteriotomy sites
 Anastomosis to femoral artery usually with 5/0 'double ended' Prolene suture
 Distal anastomosis usually using 6/0 'double ended' Prolene

Distal disease
 Femoro-distal bypass surgery takes longer to perform, is more technically
challenging and has higher failure rates.
 In elderly diabetic patients with poor runoff a primary amputation may well be
a safer and more effective option. There is no point in embarking on this type
of surgery in patients who are wheelchair bound.
 In femorodistal bypasses vein gives superior outcomes to PTFE.

Rules

 Vein mapping 1st to see whether there is suitable vein (the preferred conduit).
Sub intimal hyperplasia occurs early when PTFE is used for the distal
anastomosis and will lead to early graft occlusion and failure.
 Essential operative procedure as for above knee fem-pop.
 If there is insufficient vein for the entire conduit then vein can be attached to
the end of the PTFE graft and then used for the distal anastomosis. This type
of 'vein boot' is technically referred to as a Miller Cuff and is associated with
better patency rates than PTFE alone.
 Remember the more distal the arterial anastomosis the lower the success rate.

References
Peach G et alDiagnosis and management of peripheral arterial disease. BMJ 2012;
345: 36-41.
A 32 year old woman attends clinic for assessment of varicose veins. She has suffered
for varicose veins for many years and can trace their development back to when she
suffered a complex tibial fracture. On examination she has marked truncal varicosities
with a long tortuous long saphenous vein. Which of the following would be the most
appropriate next step?

A. Arrange a venogram

B. Arrange a venous duplex scan

C. List her for a trendelenberg procedure

D. List her for injection foam sclerotherapy

E. List her for multiple avulsion phlebectomies

This lady is likely to have deep venous incompetence as she will have been
immobilised for her tibial fracture and may well have had a DVT. A duplex scan is
mandatory prior to any form of surgical intervention. A venogram would provide
similar information but is more invasive.

Chronic venous insufficiency and Varicose veins


Wide spectrum of disease ranging from minor cosmetic problem through to ulceration
and disability. It is commoner in women than men and is worse during pregnancy.

 Defined as saccular dilatation of veins (WHO)

The veins of the lower limb consist of an interconnected network of superficial and
deep venous systems. Varices occur because of localised weakness in the vein wall
resulting in dilatation and reflux of blood due to non union of valve cusps.

 Histology: fibrous scar tissue dividing smooth muscle within media in the
vessel wall

Tissue damage in chronic venous insufficiency occurs because of perivascular


cytokine leakage resulting in localised tissue damage coupled with impaired
lymphatic flow.

 Affected veins: normally long and short saphenous veins

Diagnosis
Typical symptoms of varicose veins include:

 Cosmetic appearance
 Aching
 Ankle swelling that worsens as the day progresses
 Episodic thrombophlebitis
 Bleeding
 Itching

Symptoms of chronic venous insufficiency include:

 Dependant leg pain


 Prominent leg swelling
 Oedema extending beyond the ankle
 Venous stasis ulcers

The typical venous stasis ulcer is:

 Located above the medial malleolus


 Indolent appearance with basal granulation tissue
 Variable degree of scarring
 Non ischaemic edges
 Haemosiderin deposition in the gaiter area (and also lipodermatosclerosis).

Differential diagnosis

 Lower limb arterial disease


 Marjolins ulcer
 Claudication
 Spinal stenosis
 Swelling due to medical causes e.g. CCF.

Exclusion of these differentials is by means of physical examination and ankle


brachial pressure index measurement.

Examination

 Assess for dilated short saphenous vein (popliteal fossa) and palpate for
saphena varix medial to the femoral artery
 Brodie-Trendelenburg test: to assess level of incompetence
 Perthes' walking test: assess if deep venous system competent

Investigation

 Doppler exam: if incompetent a biphasic signal due to retrograde flow is


detected
 Duplex scanning: to ensure patent deep venous system (do if DVT or trauma)

All patients should have a Doppler assessment to assess for venous reflux and should
be classified as having uncomplicated varicose veins or varicose veins with associated
chronic venous insufficiency. In the history establishing a previous thrombotic event
(DVT/ lower limb fracture) is important and patients with such a history and all who
have evidence of chronic venous insufficiency should have a duplex scan performed.

Owing to litigation patients with saphenopopliteal incompetence should have a duplex


scan performed and the site marked by scan on the day of surgery.

Treatment
Indications for surgery:

 Cosmetic: majority
 Lipodermatosclerosis causing venous ulceration
 Recurrent superficial thrombophlebitis
 Bleeding from ruptured varix

Condition Therapy
Minor varicose veins - Reassure/ cosmetic therapy
no complications
Symptomatic In those without deep venous insufficiency options include
uncomplicated varicose foam sclerotherapy, saphenofemoral / popliteal disconnection,
veins stripping and avulsions, compression stockings
Varicose veins with Therapy as above (if compression minimum is formal class I
skin changes stockings)
Chronic venous Class 2-3 compression stockings (ensure no arterial disease).
insufficiency or ulcers

 Application of formal compression stockings (usually class II/III)

In patients who have suffered ulceration, compression stockings should be worn long
term. Where ulceration is present and established saphenofemoral reflux exists this
should be addressed surgically for durable relief of symptoms, either at the outset or
following ulcer healing.

 Injection sclerotherapy (5% Ethanolamine oleate), foam is increasingly


popular, though transient blindness has been reported. Endo venous laser
therapy is another minimally invasive option
 Sapheno-femoral or sapheno-popliteal ligation, in the case of the LSV
stripping and multiple phlebectomies

Trendelenburg procedure (sapheno-femoral junction ligation)

 Head tilt 15 degrees and legs abducted


 Oblique incision 1cm medial from artery
 Tributaries ligated (Superficial circumflex iliac vein, Superficial inferior
epigastric vein, Superficial and deep external pudendal vein)
 SF junction double ligated
 Saphenous vein stripped to level of knee/upper calf. NB increased risk of
saphenous neuralgia if stripped more distally

A 32 year old woman attends clinic for assessment of varicose veins. She has suffered
for varicose veins for many years and can trace their development back to when she
suffered a complex tibial fracture. On examination she has marked truncal varicosities
with a long tortuous long saphenous vein. Which of the following would be the most
appropriate next step?

A. Arrange a venogram

B. Arrange a venous duplex scan

C. List her for a trendelenberg procedure


D. List her for injection foam sclerotherapy

E. List her for multiple avulsion phlebectomies

This lady is likely to have deep venous incompetence as she will have been
immobilised for her tibial fracture and may well have had a DVT. A duplex scan is
mandatory prior to any form of surgical intervention. A venogram would provide
similar information but is more invasive.

Chronic venous insufficiency and Varicose veins

Wide spectrum of disease ranging from minor cosmetic problem through to ulceration
and disability. It is commoner in women than men and is worse during pregnancy.

 Defined as saccular dilatation of veins (WHO)

The veins of the lower limb consist of an interconnected network of superficial and
deep venous systems. Varices occur because of localised weakness in the vein wall
resulting in dilatation and reflux of blood due to non union of valve cusps.

 Histology: fibrous scar tissue dividing smooth muscle within media in the
vessel wall

Tissue damage in chronic venous insufficiency occurs because of perivascular


cytokine leakage resulting in localised tissue damage coupled with impaired
lymphatic flow.

 Affected veins: normally long and short saphenous veins

Diagnosis
Typical symptoms of varicose veins include:

 Cosmetic appearance
 Aching
 Ankle swelling that worsens as the day progresses
 Episodic thrombophlebitis
 Bleeding
 Itching

Symptoms of chronic venous insufficiency include:

 Dependant leg pain


 Prominent leg swelling
 Oedema extending beyond the ankle
 Venous stasis ulcers

The typical venous stasis ulcer is:

 Located above the medial malleolus


 Indolent appearance with basal granulation tissue
 Variable degree of scarring
 Non ischaemic edges
 Haemosiderin deposition in the gaiter area (and also lipodermatosclerosis).

Differential diagnosis

 Lower limb arterial disease


 Marjolins ulcer
 Claudication
 Spinal stenosis
 Swelling due to medical causes e.g. CCF.

Exclusion of these differentials is by means of physical examination and ankle


brachial pressure index measurement.

Examination

 Assess for dilated short saphenous vein (popliteal fossa) and palpate for
saphena varix medial to the femoral artery
 Brodie-Trendelenburg test: to assess level of incompetence
 Perthes' walking test: assess if deep venous system competent

Investigation

 Doppler exam: if incompetent a biphasic signal due to retrograde flow is


detected
 Duplex scanning: to ensure patent deep venous system (do if DVT or trauma)

All patients should have a Doppler assessment to assess for venous reflux and should
be classified as having uncomplicated varicose veins or varicose veins with associated
chronic venous insufficiency. In the history establishing a previous thrombotic event
(DVT/ lower limb fracture) is important and patients with such a history and all who
have evidence of chronic venous insufficiency should have a duplex scan performed.

Owing to litigation patients with saphenopopliteal incompetence should have a duplex


scan performed and the site marked by scan on the day of surgery.
Treatment
Indications for surgery:

 Cosmetic: majority
 Lipodermatosclerosis causing venous ulceration
 Recurrent superficial thrombophlebitis
 Bleeding from ruptured varix

Condition Therapy
Minor varicose veins - Reassure/ cosmetic therapy
no complications
Symptomatic In those without deep venous insufficiency options include
uncomplicated varicose foam sclerotherapy, saphenofemoral / popliteal disconnection,
veins stripping and avulsions, compression stockings
Varicose veins with Therapy as above (if compression minimum is formal class I
skin changes stockings)
Chronic venous Class 2-3 compression stockings (ensure no arterial disease).
insufficiency or ulcers

 Application of formal compression stockings (usually class II/III)

In patients who have suffered ulceration, compression stockings should be worn long
term. Where ulceration is present and established saphenofemoral reflux exists this
should be addressed surgically for durable relief of symptoms, either at the outset or
following ulcer healing.

 Injection sclerotherapy (5% Ethanolamine oleate), foam is increasingly


popular, though transient blindness has been reported. Endo venous laser
therapy is another minimally invasive option
 Sapheno-femoral or sapheno-popliteal ligation, in the case of the LSV
stripping and multiple phlebectomies

Trendelenburg procedure (sapheno-femoral junction ligation)

 Head tilt 15 degrees and legs abducted


 Oblique incision 1cm medial from artery
 Tributaries ligated (Superficial circumflex iliac vein, Superficial inferior
epigastric vein, Superficial and deep external pudendal vein)
 SF junction double ligated
 Saphenous vein stripped to level of knee/upper calf. NB increased risk of
saphenous neuralgia if stripped more distally

A 32 year old woman attends clinic for assessment of varicose veins. She has suffered
for varicose veins for many years and can trace their development back to when she
suffered a complex tibial fracture. On examination she has marked truncal varicosities
with a long tortuous long saphenous vein. Which of the following would be the most
appropriate next step?

A. Arrange a venogram

B. Arrange a venous duplex scan

C. List her for a trendelenberg procedure

D. List her for injection foam sclerotherapy

E. List her for multiple avulsion phlebectomies

This lady is likely to have deep venous incompetence as she will have been
immobilised for her tibial fracture and may well have had a DVT. A duplex scan is
mandatory prior to any form of surgical intervention. A venogram would provide
similar information but is more invasive.

Chronic venous insufficiency and Varicose veins

Wide spectrum of disease ranging from minor cosmetic problem through to ulceration
and disability. It is commoner in women than men and is worse during pregnancy.

 Defined as saccular dilatation of veins (WHO)

The veins of the lower limb consist of an interconnected network of superficial and
deep venous systems. Varices occur because of localised weakness in the vein wall
resulting in dilatation and reflux of blood due to non union of valve cusps.

 Histology: fibrous scar tissue dividing smooth muscle within media in the
vessel wall

Tissue damage in chronic venous insufficiency occurs because of perivascular


cytokine leakage resulting in localised tissue damage coupled with impaired
lymphatic flow.

 Affected veins: normally long and short saphenous veins

Diagnosis
Typical symptoms of varicose veins include:

 Cosmetic appearance
 Aching
 Ankle swelling that worsens as the day progresses
 Episodic thrombophlebitis
 Bleeding
 Itching

Symptoms of chronic venous insufficiency include:

 Dependant leg pain


 Prominent leg swelling
 Oedema extending beyond the ankle
 Venous stasis ulcers

The typical venous stasis ulcer is:

 Located above the medial malleolus


 Indolent appearance with basal granulation tissue
 Variable degree of scarring
 Non ischaemic edges
 Haemosiderin deposition in the gaiter area (and also lipodermatosclerosis).

Differential diagnosis

 Lower limb arterial disease


 Marjolins ulcer
 Claudication
 Spinal stenosis
 Swelling due to medical causes e.g. CCF.

Exclusion of these differentials is by means of physical examination and ankle


brachial pressure index measurement.

Examination

 Assess for dilated short saphenous vein (popliteal fossa) and palpate for
saphena varix medial to the femoral artery
 Brodie-Trendelenburg test: to assess level of incompetence
 Perthes' walking test: assess if deep venous system competent

Investigation

 Doppler exam: if incompetent a biphasic signal due to retrograde flow is


detected
 Duplex scanning: to ensure patent deep venous system (do if DVT or trauma)
All patients should have a Doppler assessment to assess for venous reflux and should
be classified as having uncomplicated varicose veins or varicose veins with associated
chronic venous insufficiency. In the history establishing a previous thrombotic event
(DVT/ lower limb fracture) is important and patients with such a history and all who
have evidence of chronic venous insufficiency should have a duplex scan performed.

Owing to litigation patients with saphenopopliteal incompetence should have a duplex


scan performed and the site marked by scan on the day of surgery.

Treatment
Indications for surgery:

 Cosmetic: majority
 Lipodermatosclerosis causing venous ulceration
 Recurrent superficial thrombophlebitis
 Bleeding from ruptured varix

Condition Therapy
Minor varicose veins - Reassure/ cosmetic therapy
no complications
Symptomatic In those without deep venous insufficiency options include
uncomplicated varicose foam sclerotherapy, saphenofemoral / popliteal disconnection,
veins stripping and avulsions, compression stockings
Varicose veins with Therapy as above (if compression minimum is formal class I
skin changes stockings)
Chronic venous Class 2-3 compression stockings (ensure no arterial disease).
insufficiency or ulcers

 Application of formal compression stockings (usually class II/III)

In patients who have suffered ulceration, compression stockings should be worn long
term. Where ulceration is present and established saphenofemoral reflux exists this
should be addressed surgically for durable relief of symptoms, either at the outset or
following ulcer healing.

 Injection sclerotherapy (5% Ethanolamine oleate), foam is increasingly


popular, though transient blindness has been reported. Endo venous laser
therapy is another minimally invasive option
 Sapheno-femoral or sapheno-popliteal ligation, in the case of the LSV
stripping and multiple phlebectomies

Trendelenburg procedure (sapheno-femoral junction ligation)

 Head tilt 15 degrees and legs abducted


 Oblique incision 1cm medial from artery
 Tributaries ligated (Superficial circumflex iliac vein, Superficial inferior
epigastric vein, Superficial and deep external pudendal vein)
 SF junction double ligated
 Saphenous vein stripped to level of knee/upper calf. NB increased risk of
saphenous neuralgia if stripped more distally

Theme: Management of abdominal aortic aneurysms

A. Immediate laparotomy

B. Immediate CT

C. Elective AAA repair

D. USS in 6 months

E. CT scan during next 4 weeks

F. Endovascular aortic aneurysm repair

G. Discharge

H. Palliate

I. None of the above

Please select the most appropriate management for the scenario given. Each option may be
used once, more than once or not at all.

1. A 66 year old man is referred via the aneurysm screening programme with an
abdominal aortic aneurysm measuring 4.4 cm. Apart from well controlled type 2 DM
he is otherwise well

USS in 6 months

At this point continue with ultrasound surveillance

2. A 72 year old man has a CT scan for abdominal discomfort and the surgeon suspects
AAA. This shows a 6.6cm aneurysm with a 3.5cm neck and it continues to involve the
right common iliac. The left iliac is occluded. He is hypertensive and has Type 2 DM
which is well controlled
Elective AAA repair

Assuming he is fit enough. This would be a typical 'open ' case as the marked iliac
disease would make EVAR difficult

3. An 89 year old man presents with hypotension and collapse and is found by the staff
in the toilet of his care home. He is moribund and unable to give a clear history. He
had suffered a cardiac arrest in the ambulance but has since been resuscitated and
now has a Bp of 95 systolic. He has an obviously palpable AAA.

You answered Immediate laparotomy

The correct answer is Palliate

He will not survive aortic surgery and whilst some may disagree, I would argue that
taking this case to theatre would be futile

Abdominal aorta aneurysm

 Abdominal aortic aneurysms are a common problem in vascular surgery.


 They may occur as either true or false aneurysm. With the former all 3 layers of the
arterial wall are involved, in the latter only a single layer of fibrous tissue forms the
aneurysm wall.
 True abdominal aortic aneurysms have an approximate incidence of 0.06 per 1000
people. They are commonest in elderly men and for this reason the UK is now
introducing the aneurysm screening program with the aim of performing an
abdominal aortic ultrasound measurement in all men aged 65 years.

Causes

 Several different groups of patients suffer from aneurysmal disease.


 The commonest group is those who suffer from standard arterial disease, i.e. Those
who are hypertensive, have diabetes and have been or are smokers.
 Other patients such as those suffering from connective tissue diseases such as
Marfan's may also develop aneurysms. In patients with abdominal aortic aneurysms
the extracellular matrix becomes disrupted with a change in the balance of collagen
and elastic fibres.

Management
 Most abdominal aortic aneurysms are an incidental finding.
 Symptoms most often relate to rupture or impending rupture.
 20% rupture anteriorly into the peritoneal cavity. Very poor prognosis.
 80% rupture posteriorly into the retroperitoneal space
 The risk of rupture is related to aneurysm size, only 2% of aneurysms measuring less
than 4cm in diameter will rupture over a 5 year period. This contrasts with 75% of
aneurysms measuring over 7cm in diameter.
 This is well explained by La Places' law which relates size to transmural pressure.
 For this reason most vascular surgeons will subject patients with an aneurysm size of
5cm or greater to CT scanning of the chest, abdomen and pelvis with the aim of
delineating anatomy and planning treatment. Depending upon co-morbidities,
surgery is generally offered once the aneurysm is between 5.5cm and 6cm.

A CT reconstruction showing an infrarenal abdominal aortic aneurysm. The walls of the sac
are calcified which may facilitate identification on plain x-rays
Image sourced from Wikipedia

Indications for surgery

 Symptomatic aneurysms (80% annual mortality if untreated)


 Increasing size above 5.5cm if asymptomatic
 Rupture (100% mortality without surgery)
Surgical procedures
Abdominal aortic aneurysm repair

Procedure:

GA
Invasive monitoring (A-line, CVP, catheter)
Incision: Midline or transverse
Bowel and distal duodenum mobilised to access aorta.
Aneurysm neck and base dissected out and prepared for cross clamp
Systemic heparinisation
Cross clamp (distal first)
Longitudinal aortotomy
Atherectomy
Deal with back bleeding from lumbar vessels and inferior mesenteric artery
Insert graft either tube or bifurcated depending upon anatomy
Suture using Prolene (3/0 for proximal , distal anastomosis suture varies according to site)
Clamps off: End tidal CO2 will rise owing to effects of reperfusion, at this point major risk of
myocardial events.
Haemostasis
Closure of aneurysm sac to minimise risk of aorto-enteric fistula
Closure: Loop 1 PDS or Prolene to abdominal wall
Skin- surgeons preference

Post operatively:

ITU (Almost all)


Greatest risk of complications following emergency repair
Complications: Embolic- gut and foot infarcts
Cardiac - owing to premorbid states, reperfusion injury and effects of cross clamp
Wound problems
Later risks related to graft- infection and aorto-enteric fistula

Special groups

Supra renal AAA


These patients will require a supra renal clamp and this carries a far higher risk of
complications and risk of renal failure.

Ruptured AAA
Preoperatively the management depends upon haemodynamic instability. In patients with
symptoms of rupture (typical pain, haemodynamic compromise and risk factors) then ideally
prompt laparotomy. In those with vague symptoms and haemodynamic stability the ideal
test is CT scan to determine whether rupture has occurred or not. Most common rupture
site is retroperitoneal 80%. These patients will tend to develop retroperitoneal haematoma.
This can be disrupted if Bp is allowed to rise too high so aim for Bp 100mmHg.
Operative details are similar to elective repair although surgery should be swift, blind
rushing often makes the situation worse. Plunging vascular clamps blindly into a pool of
blood at the aneurysm neck carries the risk of injury the vena cava that these patients do not
withstand. Occasionally a supracoeliac clamp is needed to effect temporary control,
although leaving this applied for more than 20 minutes tends to carry a dismal outcome.

EVAR
Increasingly patients are now being offered endovascular aortic aneurysm repair. This is
undertaken by surgeons and radiologists working jointly. The morphology of the aneurysm is
important and not all are suitable. Here is a typical list of those features favoring a suitable
aneurysm:

 Long neck
 Straight iliac vessels
 Healthy groin vessels

Clearly few AAA patients possess the above and compromise has to be made. The use of
fenestrated grafts can allow supra renal AAA to be treated.

Procedure:

GA
Radiology or theatre
Bilateral groin incisions
Common femoral artery dissected out
Heparinisation
Arteriotomy and insertion of guide wire
Dilation of arteriotomy
Insertion of EVAR Device
Once in satisfactory position it is released
Arteriotomy closed once check angiogram shows good position and no endoleak

Complications:

Endoleaks depending upon site are either Type I or 2. These may necessitate re-intervention
and all EVAR patients require follow up . Details are not needed for MRCS.

References
A reasonable review is provided by:
Sakalihasan N, Limet R, Defawe O. Abdominal aortic aneurysm. Lancet 2005 (365):1577-
1589
During short saphenous vein surgery for varicose veins which of the following nerves
is particularly at risk?

A. Sural nerve

B. Popliteal nerve

C. Tibial nerve

D. Femoral nerve

E. Saphenous nerve

Saphenous vein

Long saphenous vein

This vein may be harvested for triple or quadruple bypass surgery

 Originates at the 1st digit where the dorsal vein merges with the dorsal venous
arch of the foot
 Passes anterior to the medial malleolus and runs up the medial side of the leg
 At the knee, it runs over the posterior border of the medial epicondyle of the
femur bone
 Then passes laterally to lie on the anterior surface of the thigh before entering
an opening in the fascia lata called the saphenous opening
 It joins with the femoral vein in the region of the femoral triangle at the
saphenofemoral junction

Tributaries

 Medial marginal
 Superficial epigastric
 Superficial iliac circumflex
 Superficial external pudendal veins

Short saphenous vein

 Originates at the 5th digit where the dorsal vein merges with the dorsal venous
arch of the foot, which attaches to the great saphenous vein.
 It passes around the lateral aspect of the foot (inferior and posterior to the
lateral malleolus) and runs along the posterior aspect of the leg (with the sural
nerve)
 Passes between the heads of the gastrocnemius muscle, and drains into the
popliteal vein, approximately at or above the level of the knee joint.

Rate question:
During short saphenous vein surgery for varicose veins which of the following nerves
is particularly at risk?

A. Sural nerve

B. Popliteal nerve

C. Tibial nerve

D. Femoral nerve

E. Saphenous nerve

Saphenous vein

Long saphenous vein

This vein may be harvested for triple or quadruple bypass surgery

 Originates at the 1st digit where the dorsal vein merges with the dorsal venous
arch of the foot
 Passes anterior to the medial malleolus and runs up the medial side of the leg
 At the knee, it runs over the posterior border of the medial epicondyle of the
femur bone
 Then passes laterally to lie on the anterior surface of the thigh before entering
an opening in the fascia lata called the saphenous opening
 It joins with the femoral vein in the region of the femoral triangle at the
saphenofemoral junction

Tributaries

 Medial marginal
 Superficial epigastric
 Superficial iliac circumflex
 Superficial external pudendal veins
Short saphenous vein

 Originates at the 5th digit where the dorsal vein merges with the dorsal venous
arch of the foot, which attaches to the great saphenous vein.
 It passes around the lateral aspect of the foot (inferior and posterior to the
lateral malleolus) and runs along the posterior aspect of the leg (with the sural
nerve)
 Passes between the heads of the gastrocnemius muscle, and drains into the
popliteal vein, approximately at or above the level of the knee joint.

A 21 year old badminton player attends A&E with a painful, swollen right arm. He is
right handed. Clinically he has dusky fingers and his upper limb pulses are present.
An axillary vein thrombosis is confirmed. What is the best acute treatment to achieve
vein patency?

A. Intravenous heparin

B. Warfarin

C. Catheter directed tPA

D. Low molecular weight heparin

E. Aspirin

Heparin and warfarin prevent propagation of the clot.

Axillary vein thrombosis

 1-2% of all deep venous thrombosis


 Primary cause is associated with trauma, thoracic outlet obstruction or
repeated effort in a dominant arm (young active individuals)
 Secondary causes include central line insertion, malignancy, pacemakers

Clinical features

 Pain and swelling (non pitting)


 Numbness
 Discolouration: mottling, dusky
 Pulses present
 Congested veins
Investigations

 FBC: viscosity, platelet function


 Clotting
 Liver function tests
 D-dimer
 Duplex scan: investigation of choice
 CT scan: thoracic outlet obstruction

Treatment

 Local catheter directed TPA


 Heparin
 Warfarin

A 23 year old man presents with a brachial artery embolus. A cervical rib is suspected as
being the underlying cause. From which of the following vertebral levels do they most often
arise?

A. C7

B. C5

C. C4

D. C3

E. C2

They usually arise from C7.

Cervical ribs

 0.2-0.4% incidence
 Consist of an anomalous fibrous band that often originates from C7 and may arc
towards, but rarely reaches the sternum
 Congenital cases may present around the third decade, some cases are reported to
occur following trauma
 Bilateral in up to 70%
 Compression of the subclavian artery may produce absent radial pulse on clinical
examination and in particular may result in a positive Adsons test (lateral flexion of
the neck away from symptomatic side and traction of the symptomatic arm- leads to
obliteration of radial pulse)
 Treatment is most commonly undertaken when there is evidence of neurovascular
compromise. A transaxillary approach is the traditional operative method for
excision

Image sourced from Wikipedia

3D reconstruction of a left-sided cervical rib

A 73 year old man with rest pain and ulceration of the foot undergoes a femoro-distal
bypass graft with a PTFE graft. At the end of the procedure there are good distal foot
pulses and a warm pink foot. Over the ensuing 6 days the foot becomes progressively
cooler and the pulses diminish. What is the most likely underlying explanation for this
process?

A. Embolus

B. Neo-intimal flap

C. Neo-intimal hyperplasia

D. Polyarteritis

E. Steal syndrome
Neo-intimal hyperplasia in distal arterial anastamoses may be reduced by use of a
Miller Cuff when PTFE is the bypass conduit.
PTFE may induce neo-intimal hyperplasia with subsequent occlusion of the distal
anastomosis. In more proximal arterial bypass surgery the process of neo-intimal
hyperplasia is not sufficient to cause anastomotic occlusion. However, distal bypasses
are at greater risk and if vein cannot be used as a conduit then the distal end of the
PTFE should anastomosed to a vein cuff to minimise the risk of neo-intimal
hyperplasia.

Anastomoses

 A wide variety of anastomoses are constructed in surgical practice. Essentially


the term refers to the restoration of luminal continuity. As such they are a
feature of both abdominal and vascular surgery.

Visceral anastomoses

For an anastomosis to heal three criteria need to be fulfilled:

 Adequate blood supply


 Mucosal apposition
 Minimal tension

When these are compromise the anastomosis may dehisce (leak). Even in the best
surgical hands some anastomoses are more prone to dehiscence than others.
Oesophageal and rectal anastomoses are more prone to leakage and reported leak rates
following oesophageal and rectal surgery can be as high as 20%. This figure includes
radiological leaks and those with a clinically significant leak will be of a lower order
of magnitude. As a rule small bowel anastomoses heal most reliably.

The decision as to how best to achieve mucosal apposition is one for each surgeon.
Some will prefer the use of stapling devices as they are quicker to use, others will
prefer to perform a sutured anastomosis. The attention to surgical technique is more
important than the method chosen and a poorly constructed stapled anastomosis in
thickened tissue is far more prone to leakage than a hand sewn anastomosis in the
same circumstances.

If an anastomosis looks unsafe then it may be best not to construct one at all. In
colonic surgery this is relatively clear cut and most surgeons would bring out an end
colostomy. In situations such as oesophageal surgery this is far more problematic and
colonic interposition may be required in this situation.

Vascular anastomoses

Most arterial surgery involving bypasses or aneurysm repairs will require construction
of an arterial anastomosis. Technique is important and for small diameter distal
arterial surgery the intimal hyperplasia resulting from a badly constructed anastomosis
may render the whole operation futile before the patient leaves hospital.
Some key points about vascular anastomoses:

 Always use non absorbable monofilament suture (e.g. Polypropylene).


 Round bodied needle.
 Correct size for anastamosis ( i.e. 6/0 prolene for bottom end of a femoro-
distal bypass).
 Suture should be continuous and from inside to outside of artery to avoid
raising an intimal flap.

Theme: Lymphoedema Management

A. Homans operation
B. Charles operation
C. Frusemide at high doses
D. Frusemide at low doses
E. Multilayer compression bandaging
F. Lymphovenous anastomosis

Please select the most appropriate management for the lymphoedema scenario given.
Each option may be used once, more than once or not at all.

8. A 52 year old lady develops lower leg swelling following redo varicose vein
surgery. There is evidence of swelling of the left leg up to the knee. The
overlying skin appears healthy.

You answered Homans operation

The correct answer is Multilayer compression bandaging

Unfortunately lymphoedema may complicate redo varicose vein surgery (in


0.5% of cases). As the presentation is mild, she should be managed using
compression hosiery. Diuretics do not help in cases of true lymphoedema and a
dramatic response suggests an alternative underlying cause.

9. A 57 year old lady has suffered from lymphoedema for many years. The left
leg is swollen to the mid thigh. Severe limb deformity has developed as a
result of process and in spite of compression hoisery. Lymphoscintography
shows no patent lymphatics in the proximal leg. The overlying skin is healthy.

You answered Lymphovenous anastomosis

The correct answer is Homans operation

Surgery is indicated in less than 10% of cases. However, severe deformity is


one of the indications for surgery. Lymphovenous anastomosis is indicated
where the proximal lymphatics are not patent. When the overlying skin is
healthy (and limb deformity a problem), a Homans procedure is a reasonable
first line operative option.

10. A 38 year old lady is troubled by lymphoedema that occurred following a


block dissection of the groin for malignant melanoma many years previously.
Despite therapy with compression bandages she has persistent lower leg
swelling impairing her activities of daily living. She has no evidence of
recurrent malignancy. Lymphoscintography demonstrates occlusion of the
groin lymphatics. However, the distal lymphatic system appears healthy.

Lymphovenous anastomosis

In young patients with proximal disease and healthy distal lymphatics a


lymphovenous anastomosis may be considered. Such cases are rare.

Lymphoedema

 Due to impaired lymphatic drainage in the presence of normal capillary


function.
 Lymphoedema causes the accumulation of protein rich fluid, subdermal
fibrosis and dermal thickening.
 Characteristically fluid is confined to the epifascial space (skin and
subcutaneous tissues); muscle compartments are free of oedema. It involves
the foot, unlike other forms of oedema. There may be a 'buffalo hump' on the
dorsum of the foot and the skin cannot be pinched due to subcutaneous
fibrosis.

Causes of lymphoedema

Primary  Congenital < 1 year: sporadic, Milroy's disease


 Onset 1-35 years: sporadic, Meige's disease
 > 35 years: Tarda

Secondary  Bacterial/fungal/parasitic infection (filariasis)


 Lymphatic malignancy
 Radiotherapy to lymph nodes
 Surgical resection lymph nodes
 DVT
 Thrombophlebitis
Indications for surgery

 Marked disability or deformity from limb swelling


 Lymphoedema caused by proximal lymphatic obstruction with patent distal
lymphatics suitable for a lymphatic drainage procedure
 Lymphocutaneous fistulae and megalymphatics

Procedures
Homans operation Reduction procedure with preservation of overlying skin (which
must be in good condition). Skin flaps are raised and the
underlying tissue excised. Limb circumference typically reduced
by a third.
Charles operation All skin an subcutaneous tissue around the calf is excised down to
the deep fascia. Split skin grafts are placed over the site. May be
performed if overlying skin is not in good condition. Larger
reduction in size than with Homans procedure.
Lymphovenous Identifiable lymphatics are anastomosed to sub dermal venules.
anastamosis Usually indicated in 2% of patients with proximal lymphatic
obstruction and normal distal lymphatics.
Which of the following is not a typical feature of a chronic venous leg ulcer?

A. Heaped raised borders if the ulcer has been present more than 5 years

B. Evidence of surrounding lipodermatosclerosis

C. Irregular shape to the ulcer

D. 20% of cases will have a previous history of deep vein thrombosis

E. Haemosiderin deposits in surrounding skin

The borders of the ulcer are often well defined even though they may be irregular. Heaped
or raised borders should raise suspicion of a marjolins ulcer.

Lower leg ulcers

Venous leg ulcers

 Most due to venous hypertension, secondary to chronic venous insufficiency (other


causes include calf pump dysfunction or neuromuscular disorders)
 Ulcers form due to capillary fibrin cuff or leucocyte sequestration
 Features of venous insufficiency include oedema, brown pigmentation,
lipodermatosclerosis, eczema
 Location above the ankle, painless
 Deep venous insufficiency is related to previous DVT and superficial venous
insufficiency is associated with varicose veins
 Doppler ultrasound looks for presence of reflux and duplex ultrasound looks at the
anatomy/ flow of the vein
 Management: 4 layer compression banding after exclusion of arterial disease or
surgery
 If fail to heal after 12 weeks or >10cm2 skin grafting may be needed

Marjolin's ulcer

Image sourced from Wikipedia

 Squamous cell carcinoma


 Occurring at sites of chronic inflammation e.g; burns, osteomyelitis after 10-20 years
 Mainly occur on the lower limb

Arterial ulcers

 Occur on the toes and heel


 Painful
 There may be areas of gangrene
 Cold with no palpable pulses
 Low ABPI measurements

Neuropathic ulcers
 Commonly over plantar surface of metatarsal head and plantar surface of hallux
 The plantar neuropathic ulcer is the condition that most commonly leads to
amputation in diabetic patients
 Due to pressure
 Management includes cushioned shoes to reduce callous formation

Pyoderma gangrenosum

Image sourced from Wikipedia

 Associated with inflammatory bowel disease/RA


 Can occur at stoma sites
 Erythematous nodules or pustules which ulcerate

Which of the following is not a typical feature of an arterial leg ulcer?

A. Well demarcated edges

B. A grey - white base to the ulcer

C. Men are affected more than women

D. Painful

E. Ankle swelling

Ankle swelling is often absent. If present it may be due to mixed arteriovenous


disease. With mixed disease the arterial component is treated first.

Peripheral vascular disease

Indications for surgery to revascularise the lower limb


 Intermittent claudication
 Critical ischaemia
 Ulceration
 Gangrene

Intermittent claudication that is not disabling may provide a relative indication, whilst
the other complaints are often absolute indications depending upon the frailty of the
patient.

Assessment

 Clinical examination
 Ankle brachial pressure index measurement
 Duplex arterial ultrasound
 Angiography (standard, CT or MRI): usually performed only if intervention
being considered.

Angioplasty
In order for angioplasty to be undertaken successfully the artery has to be accessible.
The lesion relatively short and reasonable distal vessel runoff. Longer lesions may be
amenable to sub-intimal angioplasty.

Surgery
Surgery will be undertaken where attempts at angioplasty have either failed or are
unsuitable. Bypass essentially involves bypassing the affected arterial segment by
utilising a graft to run from above the disease to below the disease. As with
angioplasty good runoff improves the outcome.

Some key concepts with bypass surgery

Superficial femoral artery occlusion to the above knee

 Angioplasty may be attempted but otherwise these patients will require a


femoro-popliteal bypass graft.
 Patency rates for Polytetrafluoroethylene (PTFE) and vein are similar, so
PTFE preferred unless co-existing infection makes use of prosthetic material
undesirable.

Procedure

 Artery dissected out, IV heparin 3,000 units given and then the vessels are
cross clamped
 Longitudinal arteriotomy
 Graft cut to size and tunneled to arteriotomy sites
 Anastomosis to femoral artery usually with 5/0 'double ended' Prolene suture
 Distal anastomosis usually using 6/0 'double ended' Prolene
Distal disease

 Femoro-distal bypass surgery takes longer to perform, is more technically


challenging and has higher failure rates.
 In elderly diabetic patients with poor runoff a primary amputation may well be
a safer and more effective option. There is no point in embarking on this type
of surgery in patients who are wheelchair bound.
 In femorodistal bypasses vein gives superior outcomes to PTFE.

Rules

 Vein mapping 1st to see whether there is suitable vein (the preferred conduit).
Sub intimal hyperplasia occurs early when PTFE is used for the distal
anastomosis and will lead to early graft occlusion and failure.
 Essential operative procedure as for above knee fem-pop.
 If there is insufficient vein for the entire conduit then vein can be attached to
the end of the PTFE graft and then used for the distal anastomosis. This type
of 'vein boot' is technically referred to as a Miller Cuff and is associated with
better patency rates than PTFE alone.
 Remember the more distal the arterial anastomosis the lower the success rate.

References
Peach G et alDiagnosis and management of peripheral arterial disease. BMJ 2012;
345: 36-41.
Theme: Lower limb ulceration

A. Mixed ulcer

B. Chronic obliterative arterial disease

C. Superficial venous insufficiency

D. Deep venous insufficiency

E. Neuropathic ulcer

F. Basal cell carcinoma

G. Squamous cell carcinoma

Please select the most likely cause of ulceration for the scenario given. Each option may be
used once, more than once or not at all.

13. A 65 year old diabetic female presents with a painless ulcer at the medial malleolus,
it has been present for the past 16 years. On examination she has evidence of truncal
varicosities and a brownish discolouration of the skin overlying the affected area.

Superficial venous insufficiency

Theme from September 2012 Exam


Venous ulcers are usually associated with features of venous insufficiency. These
include haemosiderin deposition and varicose veins. Neuropathic ulcers will tend to
present at sites of pressure, which is not typically at the medial malleolus.

14. A 71 year old man presents with a painful lower calf ulcer, mild pitting oedema and
an ABPI of 0.3.

You answered Mixed ulcer

The correct answer is Chronic obliterative arterial disease

Painful ulcers associated with a low ABPI are usually arterial in nature. The question
does not indicate that features of chronic venous insufficiency are present. Patients
may have mild pitting oedema as many vascular patients will also have ischaemic
heart disease and elevated right heart pressures. The absence of more compelling
signs of venous insufficiency makes a mixed ulcer less likely.

15. A 79 year old retired teacher has had an ulcer for 15 years. It is at the medial
malleolus and has associated lipodermatosclerosis of the lower limb. The ulcer base
is heaped up and irregular.

Squamous cell carcinoma

If after many years an ulcer becomes heaped up and irregular, with rolled edges then
suspect a
squamous cell carcinoma.

Lower leg ulcers

Venous leg ulcers


 Most due to venous hypertension, secondary to chronic venous insufficiency (other
causes include calf pump dysfunction or neuromuscular disorders)
 Ulcers form due to capillary fibrin cuff or leucocyte sequestration
 Features of venous insufficiency include oedema, brown pigmentation,
lipodermatosclerosis, eczema
 Location above the ankle, painless
 Deep venous insufficiency is related to previous DVT and superficial venous
insufficiency is associated with varicose veins
 Doppler ultrasound looks for presence of reflux and duplex ultrasound looks at the
anatomy/ flow of the vein
 Management: 4 layer compression banding after exclusion of arterial disease or
surgery
 If fail to heal after 12 weeks or >10cm2 skin grafting may be needed

Marjolin's ulcer

Image sourced from Wikipedia

 Squamous cell carcinoma


 Occurring at sites of chronic inflammation e.g; burns, osteomyelitis after 10-20 years
 Mainly occur on the lower limb

Arterial ulcers

 Occur on the toes and heel


 Painful
 There may be areas of gangrene
 Cold with no palpable pulses
 Low ABPI measurements
Neuropathic ulcers

 Commonly over plantar surface of metatarsal head and plantar surface of hallux
 The plantar neuropathic ulcer is the condition that most commonly leads to
amputation in diabetic patients
 Due to pressure
 Management includes cushioned shoes to reduce callous formation

Pyoderma gangrenosum

Image sourced from Wikipedia

 Associated with inflammatory bowel disease/RA


 Can occur at stoma sites
 Erythematous nodules or pustules which ulcerate

A 66 year old man is admitted with severe angina. There is a lesion of the proximal
left anterior descending coronary artery. Which of the following would be the most
suitable conduit for bypass?

A. Long saphenous vein

B. Short saphenous vein

C. Cephalic vein

D. Internal mammary artery

E. Thoraco-acromial artery

The internal mammary artery is an excellent conduit for coronary artery bypass. It has
better long term patency rates than venous grafts. The thoraco-acromial artery is
seldom used.

Cardiopulmonary bypass

Indications for surgery

 Left main stem stenosis or equivalent (proximal LAD and proximal


circumflex)
 Triple vessel disease
 Diffuse disease unsuitable for PCI

The guidelines state that CABG is the preferred treatment in high-risk patients with
severe ventricular dysfunction or diabetes mellitus.

Technique
General anaesthesia
Central and arterial lines
Midline sternotomy or left sub mammary incision
Aortic root and pericardium dissected
Heart inspected

Bypass grafting may be performed using a cardiopulmonary bypass circuit with


cardiac arrest or using a number of novel 'off pump' techniques.

Procedure cardiopulmonary bypass

 Aortic root cannulated


 Right atrial cannula
 Circuit primed and patient fully heparinised (30,000 Units unfractionated
heparin) as the circuit is highly thrombogenic
 Flow established through circuit
 Aortic cross clamp applied
 Cardioplegia solution instilled into the aortic root below cross clamp
 Heart now asystolic and ready for surgery

Off pump techniques are evolving on a constant basis and details are beyond the
scope of the MRCS.

Conduits for bypass


> Internal mammary artery is best. Use of both is associated with increased risk of
sternal wound dehiscence. However, many surgeons will use both especially for redo
surgery.
> Radial artery harvested from forearm. Ensure ulnar collateral working first!
> Reversed long saphenous vein grafts
Typically anastamosed using 7/0-8/0 prolene sutures (distally) and 6/0 prolene for top
end.
Once flow established
Anticoagulation reversed using protamine
Patient is taken off bypass
Inotropes given if needed
Sternum closed using sternal closure device or stainless steel wire

Complications

 Post perfusion syndrome: transient cognitive impairment


 Non union of the sternum; due to loss of the internal thoracic artery
 Myocardial infarction
 Late graft stenosis
 Acute renal failure
 Stroke
 Gastrointestinal

Perioperative risk is quantified using the Parsonnet and Euroscores and unit outcomes
are audited using this data.

Reference
Eagle KA, Guyton RA, Davidoff R, et al: ACC/AHA 2004 guideline update for
coronary artery bypass graft surgery: A report of the American College of
Cardiology/American Heart Association Task Force on Practice Guidelines
(Committee to Revise the 1999 Guidelines for Coronary Artery Bypass Graft
Surgery). Circulation 2004; 110
Concerning proximal aortic dissection (Debakey types 1 and 2/ Stanford type A)
which statement is false?

A. The intimal tear is typically >50% of the aortic circumference.

B. It is usually treated using an endovascular approach.

C. They have a 50% mortality in the first 2 days.

D. Arch reconstructions may require deep hypothermic circulatory arrest.

E. Target systolic pressure of <110mmHg should be maintained.

Usually open surgery is required for these lesions as customised grafts are not usually
available for this type of repair yet.

Aortic dissection

 More common than rupture of the abdominal aorta


 33% of patients die within the first 24 hours, and 50% die within 48 hours if
no treatment received
 Associated with hypertension
 Features of aortic dissection: tear in the intimal layer, followed by formation
and propagation of a subintimal hematoma. Cystic medial necrosis (Marfan's)
 Most common site of dissection: 90% occurring within 10 centimetres of the
aortic valve

Stanford Classification
Type Location Treatment
A Ascending aorta/ aortic root Surgery- aortic root replacement
B Descending aorta Medical therapy with antihypertensives

DeBakey classification
Type Site affected
I Ascending aorta, aortic arch, descending aorta
II Ascending aorta only
III Descending aorta distal to left subclavian artery

Clinical features

 Tearing, sudden onset chest pain (painless 10%)


 Hypertension or Hypotension
 A blood pressure difference greater than 20 mm Hg
 Neurologic deficits (20%)

Investigations

 CXR: widened mediastinum, abnormal aortic knob, ring sign, deviation


trachea/oesophagus
 CT (spiral)
 MRI
 Angiography (95% of patients diagnosed)

Management

 Beta-blockers: aim HR 60-80 bpm and systolic BP 100-120 mm Hg.


 Urgent surgical intervention: type A dissections. This will usually involve
aortic root replacement.

A 67 year old male is diagnosed as having a 7cm infra renal abdominal aortic aneurysm.
What is the likely risk of rupture over the next 5 years?
A. <10%

B. 20%

C. 25%

D. 75%

E. 35%

Risks of abdominal aortic aneurysm


rupture (over 5 years):

 5-5.9cm = 25%
 6-6.9cm= 35%
 7cm and over = 75%

Aneuryms greater than 5cm in diameter on USS should be formally assessed using CT
scanning with arterial phases to delineate anatomy and facilitate surgical planning.

Abdominal aorta aneurysm

 Abdominal aortic aneurysms are a common problem in vascular surgery.


 They may occur as either true or false aneurysm. With the former all 3 layers of the
arterial wall are involved, in the latter only a single layer of fibrous tissue forms the
aneurysm wall.
 True abdominal aortic aneurysms have an approximate incidence of 0.06 per 1000
people. They are commonest in elderly men and for this reason the UK is now
introducing the aneurysm screening program with the aim of performing an
abdominal aortic ultrasound measurement in all men aged 65 years.

Causes

 Several different groups of patients suffer from aneurysmal disease.


 The commonest group is those who suffer from standard arterial disease, i.e. Those
who are hypertensive, have diabetes and have been or are smokers.
 Other patients such as those suffering from connective tissue diseases such as
Marfan's may also develop aneurysms. In patients with abdominal aortic aneurysms
the extracellular matrix becomes disrupted with a change in the balance of collagen
and elastic fibres.
Management

 Most abdominal aortic aneurysms are an incidental finding.


 Symptoms most often relate to rupture or impending rupture.
 20% rupture anteriorly into the peritoneal cavity. Very poor prognosis.
 80% rupture posteriorly into the retroperitoneal space
 The risk of rupture is related to aneurysm size, only 2% of aneurysms measuring less
than 4cm in diameter will rupture over a 5 year period. This contrasts with 75% of
aneurysms measuring over 7cm in diameter.
 This is well explained by La Places' law which relates size to transmural pressure.
 For this reason most vascular surgeons will subject patients with an aneurysm size of
5cm or greater to CT scanning of the chest, abdomen and pelvis with the aim of
delineating anatomy and planning treatment. Depending upon co-morbidities,
surgery is generally offered once the aneurysm is between 5.5cm and 6cm.

A CT reconstruction showing an infrarenal abdominal aortic aneurysm. The walls of the sac
are calcified which may facilitate identification on plain x-rays
Image sourced from Wikipedia

Indications for surgery

 Symptomatic aneurysms (80% annual mortality if untreated)


 Increasing size above 5.5cm if asymptomatic
 Rupture (100% mortality without surgery)
Surgical procedures
Abdominal aortic aneurysm repair

Procedure:

GA
Invasive monitoring (A-line, CVP, catheter)
Incision: Midline or transverse
Bowel and distal duodenum mobilised to access aorta.
Aneurysm neck and base dissected out and prepared for cross clamp
Systemic heparinisation
Cross clamp (distal first)
Longitudinal aortotomy
Atherectomy
Deal with back bleeding from lumbar vessels and inferior mesenteric artery
Insert graft either tube or bifurcated depending upon anatomy
Suture using Prolene (3/0 for proximal , distal anastomosis suture varies according to site)
Clamps off: End tidal CO2 will rise owing to effects of reperfusion, at this point major risk of
myocardial events.
Haemostasis
Closure of aneurysm sac to minimise risk of aorto-enteric fistula
Closure: Loop 1 PDS or Prolene to abdominal wall
Skin- surgeons preference

Post operatively:

ITU (Almost all)


Greatest risk of complications following emergency repair
Complications: Embolic- gut and foot infarcts
Cardiac - owing to premorbid states, reperfusion injury and effects of cross clamp
Wound problems
Later risks related to graft- infection and aorto-enteric fistula

Special groups

Supra renal AAA


These patients will require a supra renal clamp and this carries a far higher risk of
complications and risk of renal failure.

Ruptured AAA
Preoperatively the management depends upon haemodynamic instability. In patients with
symptoms of rupture (typical pain, haemodynamic compromise and risk factors) then ideally
prompt laparotomy. In those with vague symptoms and haemodynamic stability the ideal
test is CT scan to determine whether rupture has occurred or not. Most common rupture
site is retroperitoneal 80%. These patients will tend to develop retroperitoneal haematoma.
This can be disrupted if Bp is allowed to rise too high so aim for Bp 100mmHg.
Operative details are similar to elective repair although surgery should be swift, blind
rushing often makes the situation worse. Plunging vascular clamps blindly into a pool of
blood at the aneurysm neck carries the risk of injury the vena cava that these patients do not
withstand. Occasionally a supracoeliac clamp is needed to effect temporary control,
although leaving this applied for more than 20 minutes tends to carry a dismal outcome.

EVAR
Increasingly patients are now being offered endovascular aortic aneurysm repair. This is
undertaken by surgeons and radiologists working jointly. The morphology of the aneurysm is
important and not all are suitable. Here is a typical list of those features favoring a suitable
aneurysm:

 Long neck
 Straight iliac vessels
 Healthy groin vessels

Clearly few AAA patients possess the above and compromise has to be made. The use of
fenestrated grafts can allow supra renal AAA to be treated.

Procedure:

GA
Radiology or theatre
Bilateral groin incisions
Common femoral artery dissected out
Heparinisation
Arteriotomy and insertion of guide wire
Dilation of arteriotomy
Insertion of EVAR Device
Once in satisfactory position it is released
Arteriotomy closed once check angiogram shows good position and no endoleak

Complications:

Endoleaks depending upon site are either Type I or 2. These may necessitate re-intervention
and all EVAR patients require follow up . Details are not needed for MRCS.

References
A reasonable review is provided by:
Sakalihasan N, Limet R, Defawe O. Abdominal aortic aneurysm. Lancet 2005 (365):1577-
1589
Theme: Venous disease
A. No further management needed
B. Injection sclerotherapy 0.5% Sodium tetradecyl sulphate
C. Injection sclerotherapy 5% phenol
D. Long saphenous vein ligation
E. Long saphenous vein stripped to the ankle
F. Long saphenous vein stripped to the knee
G. Doppler scan
H. Duplex scan

Please select the most appropriate management plan for the scenario given. Each
option may be used once, more than once or not at all.

19. A 42 year old teacher presents with an ulcer associated with varicose veins in
the long saphenous vein territory. Apart from a DVT 1 year ago, she has no
other past medical history.

Duplex scan

This patient needs a duplex scan to assess the patency of her deep venous
system before surgery can be undertaken. Other indications for duplex scan
include recurrent varicose veins or complications.

20. A 42 year old accountant presents with thrombophlebitis of a long standing


varicosity of the inner thigh. Doppler and clinical assessment demonstrate
saphenofemoral junction incompetence.

You answered Long saphenous vein ligation

The correct answer is Long saphenous vein stripped to the knee

Long standing varicose veins with complications such as thrombophlebitis and


demonstrated valvular incompetence should receive surgery. Ligation was
previously popular but has higher long term recurrence rates. There is
increased risk of saphenous neuralgia if they are stripped to the ankle.

21. A 28 year old lady presents with increasing aching discomfort from a
varicosity below the knee. On examination she has a single large truncal
varicosity in the area, doppler assessment shows competent sapheno-femoral
and sapheno-popliteal junctions.

Injection sclerotherapy 0.5% Sodium tetradecyl sulphate

This lady has a single varicosity. Sclerotherapy (probably with foam) would be
the ideal management. A simple avulsion under anaethesia is an alternative.
These are best treated with injection sclerotherapy. 5% phenol is normally
used as a sclerosing agent in haemorrhoids!

Chronic venous insufficiency and Varicose veins

Wide spectrum of disease ranging from minor cosmetic problem through to ulceration
and disability. It is commoner in women than men and is worse during pregnancy.

 Defined as saccular dilatation of veins (WHO)

The veins of the lower limb consist of an interconnected network of superficial and
deep venous systems. Varices occur because of localised weakness in the vein wall
resulting in dilatation and reflux of blood due to non union of valve cusps.

 Histology: fibrous scar tissue dividing smooth muscle within media in the
vessel wall

Tissue damage in chronic venous insufficiency occurs because of perivascular


cytokine leakage resulting in localised tissue damage coupled with impaired
lymphatic flow.

 Affected veins: normally long and short saphenous veins

Diagnosis
Typical symptoms of varicose veins include:

 Cosmetic appearance
 Aching
 Ankle swelling that worsens as the day progresses
 Episodic thrombophlebitis
 Bleeding
 Itching

Symptoms of chronic venous insufficiency include:

 Dependant leg pain


 Prominent leg swelling
 Oedema extending beyond the ankle
 Venous stasis ulcers
The typical venous stasis ulcer is:

 Located above the medial malleolus


 Indolent appearance with basal granulation tissue
 Variable degree of scarring
 Non ischaemic edges
 Haemosiderin deposition in the gaiter area (and also lipodermatosclerosis).

Differential diagnosis

 Lower limb arterial disease


 Marjolins ulcer
 Claudication
 Spinal stenosis
 Swelling due to medical causes e.g. CCF.

Exclusion of these differentials is by means of physical examination and ankle


brachial pressure index measurement.

Examination

 Assess for dilated short saphenous vein (popliteal fossa) and palpate for
saphena varix medial to the femoral artery
 Brodie-Trendelenburg test: to assess level of incompetence
 Perthes' walking test: assess if deep venous system competent

Investigation

 Doppler exam: if incompetent a biphasic signal due to retrograde flow is


detected
 Duplex scanning: to ensure patent deep venous system (do if DVT or trauma)

All patients should have a Doppler assessment to assess for venous reflux and should
be classified as having uncomplicated varicose veins or varicose veins with associated
chronic venous insufficiency. In the history establishing a previous thrombotic event
(DVT/ lower limb fracture) is important and patients with such a history and all who
have evidence of chronic venous insufficiency should have a duplex scan performed.

Owing to litigation patients with saphenopopliteal incompetence should have a duplex


scan performed and the site marked by scan on the day of surgery.

Treatment
Indications for surgery:
 Cosmetic: majority
 Lipodermatosclerosis causing venous ulceration
 Recurrent superficial thrombophlebitis
 Bleeding from ruptured varix

Condition Therapy
Minor varicose veins - Reassure/ cosmetic therapy
no complications
Symptomatic In those without deep venous insufficiency options include
uncomplicated varicose foam sclerotherapy, saphenofemoral / popliteal disconnection,
veins stripping and avulsions, compression stockings
Varicose veins with Therapy as above (if compression minimum is formal class I
skin changes stockings)
Chronic venous Class 2-3 compression stockings (ensure no arterial disease).
insufficiency or ulcers

 Application of formal compression stockings (usually class II/III)

In patients who have suffered ulceration, compression stockings should be worn long
term. Where ulceration is present and established saphenofemoral reflux exists this
should be addressed surgically for durable relief of symptoms, either at the outset or
following ulcer healing.

 Injection sclerotherapy (5% Ethanolamine oleate), foam is increasingly


popular, though transient blindness has been reported. Endo venous laser
therapy is another minimally invasive option
 Sapheno-femoral or sapheno-popliteal ligation, in the case of the LSV
stripping and multiple phlebectomies

Trendelenburg procedure (sapheno-femoral junction ligation)

 Head tilt 15 degrees and legs abducted


 Oblique incision 1cm medial from artery
 Tributaries ligated (Superficial circumflex iliac vein, Superficial inferior
epigastric vein, Superficial and deep external pudendal vein)
 SF junction double ligated
 Saphenous vein stripped to level of knee/upper calf. NB increased risk of
saphenous neuralgia if stripped more distally

A 24-year-old female is referred to the acute surgical team as she is noted to have an
absent left radial pulse. Apart from some dizziness and lethargy, the patient does not
have any features suggestive of an acute ischaemic limb. Blood tests are as follows:

Na+ 136 mmol/l


K+ 4.1 mmol/l
Urea 2.3 mmol/l
Creatinine 77 µmol/l
 
ESR 66 mm/hr

What is the most likely diagnosis?

A. Turner's syndrome

B. Takayasu's arteritis

C. Kawasaki disease

D. Coarctation of the aorta

E. Breast carcinoma with local spread

Takayasu's arteritis

Takayasu's arteritis is a large vessel vasculitis. It typically causes occlusion of the


aorta and questions commonly refer to an absent limb pulse. It is more common in
females and Asian people

Associations

 renal artery stenosis

Management

 steroids

Theme: Leg swelling

A. Milroy's disease
B. Meige's disease
C. Lymphoedema tarda
D. Filariasis
E. Tuberculosis
F. Locally advanced bladder carcinoma
G. Malaria

Which is the most likely diagnosis for the scenario given? Each option may be used
once, more than once or not at all.
23. The medical team refer a 72 year old lady with a bilateral swollen legs. Deep
vein thrombosis has been excluded and there is no response to diuretics. On
further questioning, the patient reveals that she was born with the swelling in
both of her legs.

Milroy's disease

Milroy's disease is present from birth and is due to failure of the lymphatic
vessels to develop. Note that Meige's disease develops AFTER birth.

24. A 52 year old woman presents with rapid swelling of the left leg. The swelling
is greater in the thigh compared to the calf.

You answered Tuberculosis

The correct answer is Locally advanced bladder carcinoma

Always consider a malignancy in an older adult with new lymphoedema in a


limb, especially if the swelling is greater proximally than distally. If
malignancy is excluded consider the diagnosis of lymphoedema tarda.

25. A 34 year old African teacher attends A&E with a swollen leg. She has been in
England for 2 weeks. She lives in an area prevalent with mosquitoes and where
there is poor sanitation.

Filariasis

Filariasis is caused by the nematode Wucheria bancrofti, which is mainly


spread by mosquito. The oedema can be gross leading to elephantitis.
Treatment is with diethylcarbamazine.

Lymphoedema

 Due to impaired lymphatic drainage in the presence of normal capillary


function.
 Lymphoedema causes the accumulation of protein rich fluid, subdermal
fibrosis and dermal thickening.
 Characteristically fluid is confined to the epifascial space (skin and
subcutaneous tissues); muscle compartments are free of oedema. It involves
the foot, unlike other forms of oedema. There may be a 'buffalo hump' on the
dorsum of the foot and the skin cannot be pinched due to subcutaneous
fibrosis.
Causes of lymphoedema

Primary  Congenital < 1 year: sporadic, Milroy's disease


 Onset 1-35 years: sporadic, Meige's disease
 > 35 years: Tarda

Secondary  Bacterial/fungal/parasitic infection (filariasis)


 Lymphatic malignancy
 Radiotherapy to lymph nodes
 Surgical resection lymph nodes
 DVT
 Thrombophlebitis

Indications for surgery

 Marked disability or deformity from limb swelling


 Lymphoedema caused by proximal lymphatic obstruction with patent distal
lymphatics suitable for a lymphatic drainage procedure
 Lymphocutaneous fistulae and megalymphatics

Procedures
Homans operation Reduction procedure with preservation of overlying skin (which
must be in good condition). Skin flaps are raised and the
underlying tissue excised. Limb circumference typically reduced
by a third.
Charles operation All skin an subcutaneous tissue around the calf is excised down to
the deep fascia. Split skin grafts are placed over the site. May be
performed if overlying skin is not in good condition. Larger
reduction in size than with Homans procedure.
Lymphovenous Identifiable lymphatics are anastomosed to sub dermal venules.
anastamosis Usually indicated in 2% of patients with proximal lymphatic
obstruction and normal distal lymphatics.

A 21 year old post man notices leg pain after 5 minutes walking during his round. It improves
3 minutes after stopping. Clinically he is noted to have reduced hair of the lower limbs and
his calf muscles appear atrophied. There is a weak popliteal pulse, but it is still present when
the knee is fully extended. What is the most likely diagnosis?

A. Occlusive arterial disease caused by atherosclerosis

B. Popliteal fossa entrapment


C. Cerebral vascular accident

D. Diabetes mellitus

E. Adductor canal compression syndrome

Adductor canal compression syndrome most commonly presents in young males and is an
important differential diagnosis in men presenting with symptoms of acute limb ischaemia
on exertion. It is caused by compression of the femoral artery by the musculotendinous
band from adductor magnus muscle.
The treatment consists of the division of the abnormal band and restoration of the arterial
circulation. Popliteal fossa entrapment is the main differential diagnosis, however the pulse
disappears when the knee is fully extended.

Adductor canal

 Also called Hunter's or subsartorial canal

 Immediately distal to the apex of the femoral triangle, lying in the middle third of
the thigh. Canal terminates at the adductor hiatus.

Borders Contents

Laterally Vastus medialis muscle Saphenous nerve

Posteriorly Adductor longus, adductor magnus Superficial femoral artery

Roof Sartorius Superficial femoral vein

In the image below the sartorius muscle is removed to expose the canal contents
Image sourced from Wikipedia

Theme: Peripheral arterial disease

A. Femoro-above knee popliteal bypass graft using PTFE


B. Femoro-above knee popliteal bypass graft using long saphenous vein
C. Femoro-distal bypass graft using PTFE
D. Femoro-distal bypass graft using PTFE with Miller Cuff
E. Femoro-distal bypass graft using long saphenous vein
F. Above knee amputation
G. Below knee amputation

Please select the most appropriate management for the vascular scenario given. Each
option may be used once, more than once or not at all.

27. A 49 year old man who smokes 25 cigarettes a day presents with critical limb
ischaemia. He has no previous history of vascular surgery. MRA shows long
superficial femoral arterial occlusion with retrograde filling of the above knee
popliteal.

You answered Femoro-above knee popliteal bypass graft using long


saphenous vein

The correct answer is Femoro-above knee popliteal bypass graft using


PTFE

This man would be suitable for above knee femoro-popliteal bypass grafting.
At this level PTFE and vein will have similar patency rates. Further arterial
surgery in this man is highly likely given his young age and so vein should be
conserved for the time being.

28. A 63 year old man who previously smoked 20 cigarettes a day and has newly
diagnosed type II diabetes. He presents with rest pain. A diagnostic angiogram
demonstrates occlusion of the distal superficial femoral artery continuing
below the knee. He has reasonable posterior tibial artery below this level
which branches to give good runoff to the foot. He has varicose veins.

Femoro-distal bypass graft using PTFE with Miller Cuff

This man needs a bypass operation. Using PTFE alone will not give a good
result as sub intimal hyperplasia will give poor outcome early. Using a vein
cuff (Miller cuff) at the end of a PTFE graft will improve the situation.

29. A wheelchair bound 78 year old women with ischaemic heart disease
secondary to long smoking history and longstanding type II diabetes presents
with rest pain and a non healing ulcer on the dorsum of her foot. Angiogram
shows reasonable superficial femoral artery and iliacs. At the level of the
popliteal there is an occlusion. Below this there is a short area of patent
posterior tibial and this reconstitutes lower down the leg to flow to the foot.

You answered Below knee amputation

The correct answer is Above knee amputation

A femoro-distal bypass graft would carry a high risk of failure and risk of peri-
operative myocardial infarct. This lady would be well suited to primary
amputation.

Peripheral vascular disease


Indications for surgery to revascularise the lower limb

 Intermittent claudication
 Critical ischaemia
 Ulceration
 Gangrene

Intermittent claudication that is not disabling may provide a relative indication, whilst
the other complaints are often absolute indications depending upon the frailty of the
patient.

Assessment

 Clinical examination
 Ankle brachial pressure index measurement
 Duplex arterial ultrasound
 Angiography (standard, CT or MRI): usually performed only if intervention
being considered.

Angioplasty
In order for angioplasty to be undertaken successfully the artery has to be accessible.
The lesion relatively short and reasonable distal vessel runoff. Longer lesions may be
amenable to sub-intimal angioplasty.

Surgery
Surgery will be undertaken where attempts at angioplasty have either failed or are
unsuitable. Bypass essentially involves bypassing the affected arterial segment by
utilising a graft to run from above the disease to below the disease. As with
angioplasty good runoff improves the outcome.

Some key concepts with bypass surgery

Superficial femoral artery occlusion to the above knee

 Angioplasty may be attempted but otherwise these patients will require a


femoro-popliteal bypass graft.
 Patency rates for Polytetrafluoroethylene (PTFE) and vein are similar, so
PTFE preferred unless co-existing infection makes use of prosthetic material
undesirable.

Procedure

 Artery dissected out, IV heparin 3,000 units given and then the vessels are
cross clamped
 Longitudinal arteriotomy
 Graft cut to size and tunneled to arteriotomy sites
 Anastomosis to femoral artery usually with 5/0 'double ended' Prolene suture
 Distal anastomosis usually using 6/0 'double ended' Prolene

Distal disease

 Femoro-distal bypass surgery takes longer to perform, is more technically


challenging and has higher failure rates.
 In elderly diabetic patients with poor runoff a primary amputation may well be
a safer and more effective option. There is no point in embarking on this type
of surgery in patients who are wheelchair bound.
 In femorodistal bypasses vein gives superior outcomes to PTFE.

Rules

 Vein mapping 1st to see whether there is suitable vein (the preferred conduit).
Sub intimal hyperplasia occurs early when PTFE is used for the distal
anastomosis and will lead to early graft occlusion and failure.
 Essential operative procedure as for above knee fem-pop.
 If there is insufficient vein for the entire conduit then vein can be attached to
the end of the PTFE graft and then used for the distal anastomosis. This type
of 'vein boot' is technically referred to as a Miller Cuff and is associated with
better patency rates than PTFE alone.
 Remember the more distal the arterial anastomosis the lower the success rate.

References
Peach G et alDiagnosis and management of peripheral arterial disease. BMJ 2012;
345: 36-41.
Which of the following is not a feature of a Charcot foot?

A. Bounding foot pulses in the early phases

B. Often occurs in the complete absence of trauma

C. Erythema of the foot in the early phase

D. Autonomic neuropathy

E. Peripheral neuropathy
Do not confuse the early phase of
Charcot foot with cellulitis

Trauma (even if only minor) is a prerequisite. Patients cannot usually recall the
traumatic event. The associated neuropathy means that patients continue to walk on
the affected foot with subsequent deformity developing over time.

Peripheral vascular disease

Indications for surgery to revascularise the lower limb

 Intermittent claudication
 Critical ischaemia
 Ulceration
 Gangrene

Intermittent claudication that is not disabling may provide a relative indication, whilst
the other complaints are often absolute indications depending upon the frailty of the
patient.

Assessment

 Clinical examination
 Ankle brachial pressure index measurement
 Duplex arterial ultrasound
 Angiography (standard, CT or MRI): usually performed only if intervention
being considered.

Angioplasty
In order for angioplasty to be undertaken successfully the artery has to be accessible.
The lesion relatively short and reasonable distal vessel runoff. Longer lesions may be
amenable to sub-intimal angioplasty.

Surgery
Surgery will be undertaken where attempts at angioplasty have either failed or are
unsuitable. Bypass essentially involves bypassing the affected arterial segment by
utilising a graft to run from above the disease to below the disease. As with
angioplasty good runoff improves the outcome.

Some key concepts with bypass surgery

Superficial femoral artery occlusion to the above knee

 Angioplasty may be attempted but otherwise these patients will require a


femoro-popliteal bypass graft.
 Patency rates for Polytetrafluoroethylene (PTFE) and vein are similar, so
PTFE preferred unless co-existing infection makes use of prosthetic material
undesirable.

Procedure
 Artery dissected out, IV heparin 3,000 units given and then the vessels are
cross clamped
 Longitudinal arteriotomy
 Graft cut to size and tunneled to arteriotomy sites
 Anastomosis to femoral artery usually with 5/0 'double ended' Prolene suture
 Distal anastomosis usually using 6/0 'double ended' Prolene

Distal disease

 Femoro-distal bypass surgery takes longer to perform, is more technically


challenging and has higher failure rates.
 In elderly diabetic patients with poor runoff a primary amputation may well be
a safer and more effective option. There is no point in embarking on this type
of surgery in patients who are wheelchair bound.
 In femorodistal bypasses vein gives superior outcomes to PTFE.

Rules

 Vein mapping 1st to see whether there is suitable vein (the preferred conduit).
Sub intimal hyperplasia occurs early when PTFE is used for the distal
anastomosis and will lead to early graft occlusion and failure.
 Essential operative procedure as for above knee fem-pop.
 If there is insufficient vein for the entire conduit then vein can be attached to
the end of the PTFE graft and then used for the distal anastomosis. This type
of 'vein boot' is technically referred to as a Miller Cuff and is associated with
better patency rates than PTFE alone.
 Remember the more distal the arterial anastomosis the lower the success rate.

References
Peach G et alDiagnosis and management of peripheral arterial disease. BMJ 2012;
345: 36-41.
A 34 year old man presents with varicose veins and it is suspected that these are part
of the Klippel-Trenaunay syndrome. Which of the following is not a characteristic of
this condition?

A. Presence of varicose veins

B. Gigantism of a limb

C. Long saphenous vein involvement

D. Port wine stains with clear borders

E. Arteriovenous fistulae
The Klippel-Trenaunay vein is a large, lateral, superficial vein sometimes seen at
birth. This vein begins in the foot or the lower leg and travels proximally until it
enters the thigh or the gluteal area. Otherwise, varicosities may not be clinically
evident until the child begins to ambulate.
Varicosities may be extensive, though they often spare the saphenous distribution.
They are seen below the knee, laterally above the knee, and occasionally in the pelvic
region. Varicosities may affect the superficial, deep, and perforating venous systems.
Surgical exploration has demonstrated atresia and agenesis of deep veins,
compression due to fibrous bands, aberrant arteries, abnormal muscles, or venous
sheaths.
Rarely, varicosities have been found in the bladder, the colon, and the pulmonary
vessels

Klippel-Trenaunay syndrome

Klippel-Trenaunay-Weber syndrome generally affects a single extremity, although


cases of multiple affected limbs have been reported. The leg is the most common site
followed by the arms, the trunk, and rarely the head and the neck

Signs and symptoms


The birth defect is diagnosed by the presence of a combination of these symptoms:

 One or more distinctive port-wine stains with sharp borders


 Varicose veins
 Hypertrophy of bony and soft tissues, that may lead to local gigantism or
shrinking.
 An improperly developed lymphatic system

In some cases, port-wine stains (capillary port wine type) may be absent. Such cases
are very rare and may be classified as "atypical Klippel-Trenaunay syndrome".

KTS can either affect blood vessels, lymph vessels, or both. The condition most
commonly presents with a mixture of the two. Those with venous involvement
experience increased pain and complications.
Theme: Diabetic foot sepsis

A. IV broad spectrum antibiotics


B. Incision and drainage of pus
C. Ray amputation
D. Below knee amputation
E. Above knee amputation
F. Vacuum Assisted Closure device (VAC)
G. Discharge home
H. Application of 4 layer bandages

Please select the most appropriate management for the scenario given. Each option
may be used once, more than once or not at all.
32. A 68 year old man with type II diabetes has a non healing ulcer following a ray
amputation 2 weeks ago. An x-ray shows no osteomyelitis and the ABPI is >1.

You answered Application of 4 layer bandages

The correct answer is Vacuum Assisted Closure device (VAC)

A VAC dressing may avoid the need for further surgery.

33. A 48 year old woman is admitted with sepsis secondary to an infected diabetic
foot ulcer. She has a necrotic and infected forefoot with necrosis of the heel.
There is a boggy indurated swelling anterior to the ankle joint. Pulses are
normal.

You answered Incision and drainage of pus

The correct answer is Below knee amputation

A below knee amputation is the best option here. The foot is non salvageable.
However, she may ambulate with a prosthesis.

34. An 84 year old lady is admitted with an infected diabetic foot. An x-ray shows
osteomyelitis of her calcaneum. She has a fixed flexion deformity of her knee,
but normal pulses.

Above knee amputation

This patient will not be able to walk with a below knee amputation, therefore
an above knee amputation would be preferable, as it guarantees better healing
the short term.

Consider above knee amputation in


patients with fixed flexion deformity.

Amputations

Amputations are indicated when the affected limb is one of the following:

 Dead non viable


 Deadly where it is posing a major threat to life
 Dead useless where it is viable but a prosthesis would be preferable
Orthopaedic surgery

 Amputation is often undertaken as an option of last resort e.g. Limb salvage


has failed and the limb is so non functional that mobility needs would be best
met with prosthesis.
 Chronic fracture non union or significant limb shortening following trauma
would fit into this category. Occasionally following major trauma a primary
amputation is preferable. This would be the case in an open fracture with
major distal neurovascular compromise and other more life threatening
injuries are present.

Vascular surgery

 The first two categories are the most prevalent.


 Diabetic foot sepsis is often a major cause of sepsis which can spread rapidly
in the presence of established peripheral vascular disease.
 As a general rule the main issue in vascular surgery is to optimise vascular
inflow prior to surgery. The more distal the planned amputation is to be, the
more important this rule becomes.
 In other situations there has been something such as an embolic event that has
not been revascularised in time. In this case the limb shows fixed mottling and
an amputation will be needed.

Types of amputations
As the vast majority of commonly performed amputations affect the lower limbs these
will be covered here.

The main categories of amputations are:

 Pelvic disarticulation (hindquarter)


 Above knee amputation
 Gritti Stokes (through knee amputation)
 Below knee amputation (using either Skew or Burgess flaps)
 Syme's amputation (through ankle)
 Amputations of mid foot and digits

Choosing a level of amputation depends on:

 The disease process being treated


 Desired functional outcome
 Co-morbidities of the patient

Above knee amputations


 Quick to perform
 Heal reliably
 Patients regain their general health quickly
 For this benefit, a functional price has to be paid and many patients over the
age of 70 will never walk on an above knee prosthesis.
 Above knee amputations use equal anterior-posterior flaps

Below knee amputations

 Technically more challenging to perform


 Heal less reliably than their above knee counterparts.
 However, many more patients are able to walk using a below knee prosthesis.
 In below knee amputations the two main flaps are Skew flaps or the Burgess

Long posterior flap. There is some evidence that Skew flaps are better
vascularised than the long posterior flap and some vascular surgeons prefer
them for this reason.

It is worth remembering that whilst it may be technically feasible to offer a below


knee amputation there may be circumstances where an above knee option is
preferable. For example, in fixed flexion deformities of the lower limb, little
functional benefit would be gained from below knee amputation surgery.
Which of the following statements related to coronary artery bypass surgery is true?

A. Late graft stenosis is mainly associated with saphenous vein grafts

B. Is indicated if there is stenosis > 70% of the right coronary artery

C. The left atrium is cannulated during the procedure

D. The CHADS score is used to assess peri operative risk

E. Cardioplegia is always undertaken at a 37 degrees

Indications are:

1. Left main stem stenosis or equivalent (proximal LAD and proximal circumflex)
2. Triple vessel disease
3. Diffuse disease unsuitable for PCI
The right atrium is cannulated. The CHADS score assesses whether a patient should
be warfarinised if they have atrial fibrillation. Cardioplegia can be undertaken at cold
or warm temperatures.

Cardiopulmonary bypass

Indications for surgery


 Left main stem stenosis or equivalent (proximal LAD and proximal
circumflex)
 Triple vessel disease
 Diffuse disease unsuitable for PCI

The guidelines state that CABG is the preferred treatment in high-risk patients with
severe ventricular dysfunction or diabetes mellitus.

Technique
General anaesthesia
Central and arterial lines
Midline sternotomy or left sub mammary incision
Aortic root and pericardium dissected
Heart inspected

Bypass grafting may be performed using a cardiopulmonary bypass circuit with


cardiac arrest or using a number of novel 'off pump' techniques.

Procedure cardiopulmonary bypass

 Aortic root cannulated


 Right atrial cannula
 Circuit primed and patient fully heparinised (30,000 Units unfractionated
heparin) as the circuit is highly thrombogenic
 Flow established through circuit
 Aortic cross clamp applied
 Cardioplegia solution instilled into the aortic root below cross clamp
 Heart now asystolic and ready for surgery

Off pump techniques are evolving on a constant basis and details are beyond the
scope of the MRCS.

Conduits for bypass


> Internal mammary artery is best. Use of both is associated with increased risk of
sternal wound dehiscence. However, many surgeons will use both especially for redo
surgery.
> Radial artery harvested from forearm. Ensure ulnar collateral working first!
> Reversed long saphenous vein grafts
Typically anastamosed using 7/0-8/0 prolene sutures (distally) and 6/0 prolene for top
end.

Once flow established


Anticoagulation reversed using protamine
Patient is taken off bypass
Inotropes given if needed
Sternum closed using sternal closure device or stainless steel wire
Complications

 Post perfusion syndrome: transient cognitive impairment


 Non union of the sternum; due to loss of the internal thoracic artery
 Myocardial infarction
 Late graft stenosis
 Acute renal failure
 Stroke
 Gastrointestinal

Perioperative risk is quantified using the Parsonnet and Euroscores and unit outcomes
are audited using this data.

Reference
Eagle KA, Guyton RA, Davidoff R, et al: ACC/AHA 2004 guideline update for
coronary artery bypass graft surgery: A report of the American College of
Cardiology/American Heart Association Task Force on Practice Guidelines
(Committee to Revise the 1999 Guidelines for Coronary Artery Bypass Graft
Surgery). Circulation 2004; 110
A 73 year old man develops sudden onset abdominal pain and collapses. On examination he
has a tender pulsatile mass in his upper abdomen. He has a blood pressure of 90/60mmHg
and pulse rate of 105 beats per minute. Which of the following intravenous fluid regimens is
most appropriate, whilst waiting for operative repair?

A. 1 Litre of pentastarch over 15 minutes

B. 1 litre of Hartmans solution over 4 hours

C. 1 litre of gelofusin over 30 minutes

D. 1 litre of Hartmans solution over 30 minutes

E. 1 litre of blood over 15 minutes

This man will have a contained haematoma and is awaiting surgery. Rapid, high volume
infusions may cause this to dislodge with disastrous consequences

Abdominal aorta aneurysm

 Abdominal aortic aneurysms are a common problem in vascular surgery.


 They may occur as either true or false aneurysm. With the former all 3 layers of the
arterial wall are involved, in the latter only a single layer of fibrous tissue forms the
aneurysm wall.
 True abdominal aortic aneurysms have an approximate incidence of 0.06 per 1000
people. They are commonest in elderly men and for this reason the UK is now
introducing the aneurysm screening program with the aim of performing an
abdominal aortic ultrasound measurement in all men aged 65 years.

Causes

 Several different groups of patients suffer from aneurysmal disease.


 The commonest group is those who suffer from standard arterial disease, i.e. Those
who are hypertensive, have diabetes and have been or are smokers.
 Other patients such as those suffering from connective tissue diseases such as
Marfan's may also develop aneurysms. In patients with abdominal aortic aneurysms
the extracellular matrix becomes disrupted with a change in the balance of collagen
and elastic fibres.

Management

 Most abdominal aortic aneurysms are an incidental finding.


 Symptoms most often relate to rupture or impending rupture.
 20% rupture anteriorly into the peritoneal cavity. Very poor prognosis.
 80% rupture posteriorly into the retroperitoneal space
 The risk of rupture is related to aneurysm size, only 2% of aneurysms measuring less
than 4cm in diameter will rupture over a 5 year period. This contrasts with 75% of
aneurysms measuring over 7cm in diameter.
 This is well explained by La Places' law which relates size to transmural pressure.
 For this reason most vascular surgeons will subject patients with an aneurysm size of
5cm or greater to CT scanning of the chest, abdomen and pelvis with the aim of
delineating anatomy and planning treatment. Depending upon co-morbidities,
surgery is generally offered once the aneurysm is between 5.5cm and 6cm.

A CT reconstruction showing an infrarenal abdominal aortic aneurysm. The walls of the sac
are calcified which may facilitate identification on plain x-rays
Image sourced from Wikipedia

Indications for surgery

 Symptomatic aneurysms (80% annual mortality if untreated)


 Increasing size above 5.5cm if asymptomatic
 Rupture (100% mortality without surgery)
Surgical procedures
Abdominal aortic aneurysm repair

Procedure:

GA
Invasive monitoring (A-line, CVP, catheter)
Incision: Midline or transverse
Bowel and distal duodenum mobilised to access aorta.
Aneurysm neck and base dissected out and prepared for cross clamp
Systemic heparinisation
Cross clamp (distal first)
Longitudinal aortotomy
Atherectomy
Deal with back bleeding from lumbar vessels and inferior mesenteric artery
Insert graft either tube or bifurcated depending upon anatomy
Suture using Prolene (3/0 for proximal , distal anastomosis suture varies according to site)
Clamps off: End tidal CO2 will rise owing to effects of reperfusion, at this point major risk of
myocardial events.
Haemostasis
Closure of aneurysm sac to minimise risk of aorto-enteric fistula
Closure: Loop 1 PDS or Prolene to abdominal wall
Skin- surgeons preference

Post operatively:

ITU (Almost all)


Greatest risk of complications following emergency repair
Complications: Embolic- gut and foot infarcts
Cardiac - owing to premorbid states, reperfusion injury and effects of cross clamp
Wound problems
Later risks related to graft- infection and aorto-enteric fistula

Special groups

Supra renal AAA


These patients will require a supra renal clamp and this carries a far higher risk of
complications and risk of renal failure.

Ruptured AAA
Preoperatively the management depends upon haemodynamic instability. In patients with
symptoms of rupture (typical pain, haemodynamic compromise and risk factors) then ideally
prompt laparotomy. In those with vague symptoms and haemodynamic stability the ideal
test is CT scan to determine whether rupture has occurred or not. Most common rupture
site is retroperitoneal 80%. These patients will tend to develop retroperitoneal haematoma.
This can be disrupted if Bp is allowed to rise too high so aim for Bp 100mmHg.
Operative details are similar to elective repair although surgery should be swift, blind
rushing often makes the situation worse. Plunging vascular clamps blindly into a pool of
blood at the aneurysm neck carries the risk of injury the vena cava that these patients do not
withstand. Occasionally a supracoeliac clamp is needed to effect temporary control,
although leaving this applied for more than 20 minutes tends to carry a dismal outcome.

EVAR
Increasingly patients are now being offered endovascular aortic aneurysm repair. This is
undertaken by surgeons and radiologists working jointly. The morphology of the aneurysm is
important and not all are suitable. Here is a typical list of those features favoring a suitable
aneurysm:

 Long neck
 Straight iliac vessels
 Healthy groin vessels

Clearly few AAA patients possess the above and compromise has to be made. The use of
fenestrated grafts can allow supra renal AAA to be treated.

Procedure:

GA
Radiology or theatre
Bilateral groin incisions
Common femoral artery dissected out
Heparinisation
Arteriotomy and insertion of guide wire
Dilation of arteriotomy
Insertion of EVAR Device
Once in satisfactory position it is released
Arteriotomy closed once check angiogram shows good position and no endoleak

Complications:

Endoleaks depending upon site are either Type I or 2. These may necessitate re-intervention
and all EVAR patients require follow up . Details are not needed for MRCS.

References
A reasonable review is provided by:
Sakalihasan N, Limet R, Defawe O. Abdominal aortic aneurysm. Lancet 2005 (365):1577-
1589
Theme: Mesenteric vascular disease
A. Abdominal ultrasound
B. Abdominal CT with venous phase contrast
C. CT angiogram
D. Magnetic resonance angiogram
E. Duplex ultrasound
F. Abdominal x-ray
G. Meseneric angiography via brachial artery

Please select the most appropriate investigation for the scenario given. Each option
may be used once, more than once or not at all.

37. A thin 72 year old lady has a 3 week history of postprandial abdominal pain
that is centrally located. She has episodic diarrhoea and occasionally has
passed blood PR. She has a history of ischaemic heart disease and marked
renal impairment from ACE inhibitor usage.

You answered Magnetic resonance angiogram

The correct answer is Duplex ultrasound

She is likely to have mesenteric vascular disease. Proximal SMA disease


would be the most serious variant. Ideally a CT angiogram would be the best
test but with her impaired renal function and low BMI, make a duplex of the
SMA is a reasonable first line investigation. Gut peristalsis may impair
acquisition of magnetic resonance images.

38. A 78 year old man develops sudden onset abdominal pain and almost
immediately afterwards passes a large amount of diarrhoea.

CT angiogram

Sudden onset of abdominal pain followed by forceful evacuation are the


classical presenting features of acute mesenteric infarction. This is best
investigated by CT angiography, which has a sensitivity of 95% for the
diagnosis.

39. A 28 year old female has suffered from abdominal pain for the past 2 weeks
since she was started on the contraceptive pill. The pain has increased
significantly over the post 10 hours and has been associated with vomiting.

You answered Abdominal x-ray

The correct answer is CT angiogram


Mesenteric venous thrombosis is the likely underlying cause and an angiogram
is the sensible step as it will also facilitate the identification of areas of
infarcted bowel , similar to that which may occur in the leg when massive
DVT is present.

Vascular investigations

Venous disease

Venous Doppler
The simplest investigation for assessment of venous junctional incompetence is a
Doppler assessment. This involves the patient standing and manual compression of
the limb distal to the junction of interest. Flow should normally occur in one direction
only. Where junctional incompetence is present reverse flow will occur and is
relatively easy to identify.

Venograms and duplex scans


Structural venous information is historically obtained using a venogram. This is an
invasive test and rarely required in modern clinical practice. The most helpful test is a
venous duplex scan which will provide information relating to flow and vessel
characteristics. Duplex is also useful in providing vein maps for bypass surgery.

Arterial disease

Ankle-brachial pressure
The ankle brachial pressure index measurement is an important investigation as it will
allow classification of the severity of the flow compromise present. False readings
may occur in those with calcified vessels such as diabetics and results in such settings
should be interpreted with caution. When auscultating the vessel note should be made
of the character of the signal. Monophasic signals are associated with a proximal
stenosis and reduction in flow. Triphasic signals provide reassurance of a healthy
vessel.

Arterial Duplex
As with the vein the duplex scan can provide a substantial amount of information
about arterial patency and flow patterns. In skilled hands they can provide insight as
to the state of proximal vessels that are anatomically inaccessible to duplex (e.g.
Iliacs). Through assessment of distal flow patterns. It is an operator dependent test.

Conventional angiogram
Vessel puncture and catheter angiography is the gold standard method of assessing
arteries. High quality information can usually be obtained. Limitations of the
technique include the risk of contrast toxicity and risks of vessel damage. Severely
calcified vessels may be difficult to puncture and in this situation a remote access site
(e.g. brachial) may be used. This technique is particularly useful in providing a distal
arterial roadmap prior to femoro-distal bypass.
CT angiography
These tests provide a considerable amount of structural and flow information. They
require contrast and thus carry the risks associated with this. They are particularly
useful in the setting of GI bleeding as they are rapidly available and can be performed
by a non vascular radiologist. However, they lack the facility for endovascular
intervention. In general they do not provide high enough resolution for distal arterial
surgery.

Magnetic resonance angiography


This has the advantage of being non-invasive and not using nephrotoxic contrast.
Movement artifact remains a problem in some sites and distal arterial resolution is
imperfect.
A 52 year old male attends the stroke unit with dizziness and vertigo while playing tennis. He
is known to have hypertension and a previous myocardial infarct. He now complains of right
arm pain. What is the most likely diagnosis?

A. Posterior circulation infarct

B. Vertebrobasilar aneurysm

C. Dissection of thoracic aorta

D. Subclavian steal syndrome

E. Left middle cerebral artery infarct

Subclavian steal syndrome characteristically presents with posterior circulation symptoms,


such as dizziness and vertigo, during exertion of an arm. There is subclavian artery steno-
occlusive disease proximal to the origin of the vertebral artery and is associated with flow
reversal in the vertebral artery. Management involves percutaneous transluminal
angioplasty or a stent.

Subclavian artery

Path

 The left subclavian comes directly off the arch of aorta


 The right subclavian arises from the brachiocephalic artery (trunk) when it bifurcates
into the subclavian and the right common carotid artery.
 From its origin, the subclavian artery travels laterally, passing between anterior and
middle scalene muscles, deep to scalenus anterior and anterior to scalenus medius.
As the subclavian artery crosses the lateral border of the first rib, it becomes the
axillary artery. At this point it is superficial and within the subclavian triangle.
Image sourced from Wikipedia

Branches

 Vertebral artery
 Internal thoracic artery
 Thyrocervical trunk
 Costocervical trunk
 Dorsal scapular artery

A 74 year old lady has a long standing venous leg ulcer overlying her medial malleolus.
Which of the following statements relating to the management of this condition is false?

A. Pentoxifylline may speed ulcer healing

B. Treatment with daily low dose flucloxacillin may speed ulcer healing

C. Multilayer bandages may provide compression equivalent to 40mmHg

D. Large ulcers may be considered for skin grafting

E. They should not be treated with compression stockings if the ankle /


brachial pressure index is 0.6

Routine use of antibiotics is not advised as this may predispose to resistant organisms.
Pentoxifylline was subjected to a Cochrane review in 2007 and shown to improve healing
rates.

Lower leg ulcers

Venous leg ulcers

 Most due to venous hypertension, secondary to chronic venous insufficiency (other


causes include calf pump dysfunction or neuromuscular disorders)
 Ulcers form due to capillary fibrin cuff or leucocyte sequestration
 Features of venous insufficiency include oedema, brown pigmentation,
lipodermatosclerosis, eczema
 Location above the ankle, painless
 Deep venous insufficiency is related to previous DVT and superficial venous
insufficiency is associated with varicose veins
 Doppler ultrasound looks for presence of reflux and duplex ultrasound looks at the
anatomy/ flow of the vein
 Management: 4 layer compression banding after exclusion of arterial disease or
surgery
 If fail to heal after 12 weeks or >10cm2 skin grafting may be needed

Marjolin's ulcer

Image sourced from Wikipedia

 Squamous cell carcinoma


 Occurring at sites of chronic inflammation e.g; burns, osteomyelitis after 10-20 years
 Mainly occur on the lower limb

Arterial ulcers
 Occur on the toes and heel
 Painful
 There may be areas of gangrene
 Cold with no palpable pulses
 Low ABPI measurements

Neuropathic ulcers

 Commonly over plantar surface of metatarsal head and plantar surface of hallux
 The plantar neuropathic ulcer is the condition that most commonly leads to
amputation in diabetic patients
 Due to pressure
 Management includes cushioned shoes to reduce callous formation

Pyoderma gangrenosum

Image sourced from Wikipedia

 Associated with inflammatory bowel disease/RA


 Can occur at stoma sites
 Erythematous nodules or pustules which ulcerate
 Theme: Investigation of vascular disease

A. Angiography
B. Arterial duplex scan
C. Arch aortogram
D. CXR
E. CT scan
F. Venous duplex scan

Please select the most appropriate investigation for the scenario given. Each
option may be used once, more than once or not at all.
42. A 22 year old professional tennis player attends A&E with a swollen painful
right arm. His fingers are dusky.

You answered Arterial duplex scan

The correct answer is Venous duplex scan

This patient has an axillary vein thrombosis. It classically presents with pain
and swelling of an effort induced limb. Duplex scan is needed to exclude a
thombus.

43. A 65 year old man presents with expressive dysphasia and left sided weakness
over 4h. His symptoms have now completely resolved.

You answered Arch aortogram

The correct answer is Arterial duplex scan

This patient has had a transient ischaemic attack. He urgently needs carotid
duplex scanning to assess if he needs a carotid endarterectomy.

44. A 65 year old man presents, for the first time, with pain at the back of his
calves when he mobilises 10 metres. He is known to have hypertension.

You answered Angiography

The correct answer is Arterial duplex scan

An arterial duplex should be performed first, before progression to an


angiography.


 Vascular investigations

Venous disease

Venous Doppler
The simplest investigation for assessment of venous junctional incompetence
is a Doppler assessment. This involves the patient standing and manual
compression of the limb distal to the junction of interest. Flow should
normally occur in one direction only. Where junctional incompetence is
present reverse flow will occur and is relatively easy to identify.

Venograms and duplex scans


Structural venous information is historically obtained using a venogram. This
is an invasive test and rarely required in modern clinical practice. The most
helpful test is a venous duplex scan which will provide information relating to
flow and vessel characteristics. Duplex is also useful in providing vein maps
for bypass surgery.

Arterial disease

Ankle-brachial pressure
The ankle brachial pressure index measurement is an important investigation
as it will allow classification of the severity of the flow compromise present.
False readings may occur in those with calcified vessels such as diabetics and
results in such settings should be interpreted with caution. When auscultating
the vessel note should be made of the character of the signal. Monophasic
signals are associated with a proximal stenosis and reduction in flow.
Triphasic signals provide reassurance of a healthy vessel.

Arterial Duplex
As with the vein the duplex scan can provide a substantial amount of
information about arterial patency and flow patterns. In skilled hands they can
provide insight as to the state of proximal vessels that are anatomically
inaccessible to duplex (e.g. Iliacs). Through assessment of distal flow patterns.
It is an operator dependent test.

Conventional angiogram
Vessel puncture and catheter angiography is the gold standard method of
assessing arteries. High quality information can usually be obtained.
Limitations of the technique include the risk of contrast toxicity and risks of
vessel damage. Severely calcified vessels may be difficult to puncture and in
this situation a remote access site (e.g. brachial) may be used. This technique
is particularly useful in providing a distal arterial roadmap prior to femoro-
distal bypass.

CT angiography
These tests provide a considerable amount of structural and flow information.
They require contrast and thus carry the risks associated with this. They are
particularly useful in the setting of GI bleeding as they are rapidly available
and can be performed by a non vascular radiologist. However, they lack the
facility for endovascular intervention. In general they do not provide high
enough resolution for distal arterial surgery.

Magnetic resonance angiography


This has the advantage of being non-invasive and not using nephrotoxic
contrast. Movement artifact remains a problem in some sites and distal arterial
resolution is imperfect.
heme: Ankle Brachial pressure index measurements

A. >1
B. 0.6- 0.8
C. 1
D. 0.4-0.6
E. <0.4

For the scenarios described below, please select the most likely ankle brachial
pressure index measurement. Each option may be used once, more than once or not at
all.

45. An 83 year old male with rest pain.

<0.4

Theme from April 2012 Exam


Rest pain is typically associated with low ABPI values.

46. A 45 year old man who develops calf pain after walking 600 yards. It resolves
during periods of rest.

0.6- 0.8

Since this is a long claudication distance it may be that only a minor lesion is
present. Whilst resting ABPI may be normal, they are usually abnormal
following exercise.

47. A 43 year old lady with long standing diabetes who complains of calf pain. It
is worse at night and during minor exercise.

>1

Diabetes may be complicated by vessel calcification and neuropathic pain.


Therefore individuals may present with pain which is atypical for claudation
both in terms of its tempo of onset and location.

Ankle-Brachial pressure index

 Measurement of ankle- brachial pressure index (ABPI) is a commonly


performed vascular investigation.
 Calculated by dividing lower limb pressure by the highest upper limb pressure.

Results of ABPI
1.2 or greater Usually due to vessel calcification
1.0- 1.2 Normal
0.8-1.0 Minor stenotic lesion
Initiate risk factor management
0.50-0.8 Moderate stenotic lesion
Consider duplex
Risk factor management
If mixed ulcers present then avoid tight compression bandages
0.5- 0.3 Likely significant stenosis
Duplex scanning to delineate lesions needed
Compression bandaging contra indicated
Less than 0.3 Indicative of critical ischaemia
Urgent detailed imaging required
Theme: Causes of ulceration

A. Marjolin's ulcer

B. Neuropathic ulcer

C. Arterial ulcer

D. Deep venous dysfunction

E. Superficial venous dysfunction

F. Rheumatoid arthritis

G. Pyoderma gangrenosum

H. Pressure ulcer

Please select the most likely cause of ulceration for the scenario given. Each option may be
used once, more than once or not at all.

48. A 62 year old diabetic man presents with long standing plantar ulcer he has clinical
evidence of a charcot foot.

You answered Rheumatoid arthritis

The correct answer is Neuropathic ulcer

Plantar ulcers in association with peripheral neuropathy are often neuropathic. They
classically occur at pressure points.

49. A 66 year old female has long standing mixed arteriovenous ulcers of the lower leg.
Over the past 6 months one of the ulcers has become much worse and despite a
number of different topical therapies is increasing in size.

Marjolin's ulcer

Marjolin's ulcer is a squamous cell carcinoma occurring at sites of chronic


inflammation or previous injury.

50. A 28 year old man undergoes a ileocaecal resection and end ileostomy for Crohn's
disease. One year later he presents with a deep painful ulcer at his stoma site.

Pyoderma gangrenosum

Pyoderma gangrenosum is associated with inflammatory bowel disease (this patient


had a stoma for crohns!). It is commonly found on lower limbs and described as
being painful, the size of an insect bite and growing. It looks like a margarita pizza
(with a red base and yellow topping!) Treatment involves steroids.

Lower leg ulcers

Venous leg ulcers

 Most due to venous hypertension, secondary to chronic venous insufficiency (other


causes include calf pump dysfunction or neuromuscular disorders)
 Ulcers form due to capillary fibrin cuff or leucocyte sequestration
 Features of venous insufficiency include oedema, brown pigmentation,
lipodermatosclerosis, eczema
 Location above the ankle, painless
 Deep venous insufficiency is related to previous DVT and superficial venous
insufficiency is associated with varicose veins
 Doppler ultrasound looks for presence of reflux and duplex ultrasound looks at the
anatomy/ flow of the vein
 Management: 4 layer compression banding after exclusion of arterial disease or
surgery
 If fail to heal after 12 weeks or >10cm2 skin grafting may be needed

Marjolin's ulcer
Image sourced from Wikipedia

 Squamous cell carcinoma


 Occurring at sites of chronic inflammation e.g; burns, osteomyelitis after 10-20 years
 Mainly occur on the lower limb

Arterial ulcers

 Occur on the toes and heel


 Painful
 There may be areas of gangrene
 Cold with no palpable pulses
 Low ABPI measurements

Neuropathic ulcers

 Commonly over plantar surface of metatarsal head and plantar surface of hallux
 The plantar neuropathic ulcer is the condition that most commonly leads to
amputation in diabetic patients
 Due to pressure
 Management includes cushioned shoes to reduce callous formation

Pyoderma gangrenosum
Image sourced from Wikipedia

 Associated with inflammatory bowel disease/RA


 Can occur at stoma sites
 Erythematous nodules or pustules which ulcerate

Theme: Causes of ulceration

A. Marjolin's ulcer

B. Neuropathic ulcer

C. Arterial ulcer

D. Deep venous dysfunction

E. Superficial venous dysfunction

F. Rheumatoid arthritis

G. Pyoderma gangrenosum

H. Pressure ulcer

Please select the most likely cause of ulceration for the scenario given. Each option may be
used once, more than once or not at all.

48. A 62 year old diabetic man presents with long standing plantar ulcer he has clinical
evidence of a charcot foot.

You answered Rheumatoid arthritis


The correct answer is Neuropathic ulcer

Plantar ulcers in association with peripheral neuropathy are often neuropathic. They
classically occur at pressure points.

49. A 66 year old female has long standing mixed arteriovenous ulcers of the lower leg.
Over the past 6 months one of the ulcers has become much worse and despite a
number of different topical therapies is increasing in size.

Marjolin's ulcer

Marjolin's ulcer is a squamous cell carcinoma occurring at sites of chronic


inflammation or previous injury.

50. A 28 year old man undergoes a ileocaecal resection and end ileostomy for Crohn's
disease. One year later he presents with a deep painful ulcer at his stoma site.

Pyoderma gangrenosum

Pyoderma gangrenosum is associated with inflammatory bowel disease (this patient


had a stoma for crohns!). It is commonly found on lower limbs and described as
being painful, the size of an insect bite and growing. It looks like a margarita pizza
(with a red base and yellow topping!) Treatment involves steroids.

Lower leg ulcers

Venous leg ulcers

 Most due to venous hypertension, secondary to chronic venous insufficiency (other


causes include calf pump dysfunction or neuromuscular disorders)
 Ulcers form due to capillary fibrin cuff or leucocyte sequestration
 Features of venous insufficiency include oedema, brown pigmentation,
lipodermatosclerosis, eczema
 Location above the ankle, painless
 Deep venous insufficiency is related to previous DVT and superficial venous
insufficiency is associated with varicose veins
 Doppler ultrasound looks for presence of reflux and duplex ultrasound looks at the
anatomy/ flow of the vein
 Management: 4 layer compression banding after exclusion of arterial disease or
surgery
 If fail to heal after 12 weeks or >10cm2 skin grafting may be needed

Marjolin's ulcer

Image sourced from Wikipedia

 Squamous cell carcinoma


 Occurring at sites of chronic inflammation e.g; burns, osteomyelitis after 10-20 years
 Mainly occur on the lower limb

Arterial ulcers

 Occur on the toes and heel


 Painful
 There may be areas of gangrene
 Cold with no palpable pulses
 Low ABPI measurements

Neuropathic ulcers

 Commonly over plantar surface of metatarsal head and plantar surface of hallux
 The plantar neuropathic ulcer is the condition that most commonly leads to
amputation in diabetic patients
 Due to pressure
 Management includes cushioned shoes to reduce callous formation
Pyoderma gangrenosum

Image sourced from Wikipedia

 Associated with inflammatory bowel disease/RA


 Can occur at stoma sites
 Erythematous nodules or pustules which ulcerate

You might also like